0% found this document useful (0 votes)
2K views106 pages

Multiple Choice Questions Variable vs. Absorption Costing

Variable costing and absorption costing are two different methods for assigning costs to products. Variable costing treats variable costs as product costs and fixed costs as period costs, while absorption costing treats both variable and fixed costs as product costs. Absorption costing results in higher inventory balances and cost of goods sold than variable costing. Variable costing is more appropriate for internal reporting and management decision making, while absorption costing is required for external financial reporting.

Uploaded by

Nicole Capundan
Copyright
© © All Rights Reserved
We take content rights seriously. If you suspect this is your content, claim it here.
Available Formats
Download as PDF, TXT or read online on Scribd
0% found this document useful (0 votes)
2K views106 pages

Multiple Choice Questions Variable vs. Absorption Costing

Variable costing and absorption costing are two different methods for assigning costs to products. Variable costing treats variable costs as product costs and fixed costs as period costs, while absorption costing treats both variable and fixed costs as product costs. Absorption costing results in higher inventory balances and cost of goods sold than variable costing. Variable costing is more appropriate for internal reporting and management decision making, while absorption costing is required for external financial reporting.

Uploaded by

Nicole Capundan
Copyright
© © All Rights Reserved
We take content rights seriously. If you suspect this is your content, claim it here.
Available Formats
Download as PDF, TXT or read online on Scribd
You are on page 1/ 106

Multiple Choice Questions Variable vs.

Absorption
Costing

VARIABLE COSTING
1
. Which of the following is a term more descriptive of the
type of cost accounting often called direct costing?
a. Out-of-pocket costing.
b. Variable costing.
c. Relevant costing.
d. Prime costing.
(Adapted)
2
. Which of the following must be known about a
production process to institute a variable costing
system?
a. The variable and fixed components of all costs
related to production.
b. The controllable and noncontrollable components of
all costs related to production.
c. Standard production rates and times for all
elements of production.
d. Contribution margin and breakeven point for all
goods in production.
(Adapted)
3
. What costs are treated as product cost under variable
costing?
a. Only direct costs.
b. Only variable production costs.
c. All variable costs.
d. All variable and fixed manufacturing costs.
(Adapted)
4
. Inventory under the variable costing method includes
a. Direct materials cost, direct labor cost, but no
factory overhead cost.
b. Direct materials cost, direct labor cost, and variable
factory overhead cost.
c. Prime cost but not conversion cost.
d. Prime cost and all conversion cost.
(Adapted)
5
. In the application of variable costing as a cost –
allocation process in manufacturing,
a. Variable direct costs are treated as period costs.

1
Chapter 6 Variable vs.
Absorption Costing
b. Nonvariable indirect costs are treated as product
costs.
c. Variable indirect costs are treated as product costs.
d. Nonvariable direct costs are treated as product
costs.
(Adapted)
6
. Cay Co.’s 1998 fixed manufacturing overhead costs
totaled P100,000, and variable selling costs totaled
P80,000. Under variable costing, how should these
costs be classified?
Period Costs Product Costs
a. P0 P180,000
b. P80,000 P100,000
c. P100,000 P 80,000
d. P180,000 P0
7
. Under the variable-costing concept, unit product cost
would most likely be increased by
a. A decrease in the remaining useful life of factory
machinery depreciated on the units-of-production
method.
b. A decrease in the number of units produced.
c. An increase in the remaining useful life of factory
machinery depreciated on the sum-of-the-year’s-
digits method.
d. An increase in the commission paid to salesmen for
each unit sold.
(Adapted)
8
. Which costing method is properly classified as to its
acceptability for both external and internal reporting?
External
Internal
Reporting
Reporting

a. Activity-based costing Yes Yes

b. Job costing No Yes


c. Variable costing Yes No
d. Process costing No Yes
(Adapted)

2
Multiple Choice Questions Variable vs. Absorption
Costing
9
. A basic tenet of variable costing is that period costs
should be currently expensed. What is the rationale
behind this procedure?
a. Period costs are uncontrollable and should not be
charged to a specific product.
b. Period cists are usually immaterial in amount, and
the cost of assigning them to specific products will
outweigh the benefits.
c. Allocation of period costs is arbitrary at best and
could lead to erroneous decisions by management.
d. Because period costs will occur whether or not
production occurs, it is improper to allocate these
costs to production and defer a current cost of
doing business.
(Adapted)
10
. Which of the following is an argument against the use of
variable costing?
a. Absorption costing overstates the balance sheet
value of inventories.
b. Variable factory overhead is a period cost.
c. Fixed factory overhead is difficult to allocate
properly.
d. Fixed factory overhead is necessary for the
production of a product.
(Adapted)
11
. In an income statement prepared as an internal report
using the variable costing method, which of the
following terms should appear?

Gross Profit Operating


(Margin) Income
a. Yes Yes
b. Yes No
c. No No
d. No Yes
(Adapted)
12
. In an income statement prepared as an internal report
using the variable costing method, variable selling and
administrative expenses are
a. Not used.

3
Chapter 6 Variable vs.
Absorption Costing
b. Treated the same as fixed selling and administrative
expenses.
c. Used in the computation of operating income but
not in the computation of the contribution margin.
d. Used in the computation of the contribution margin.
(Adapted)
13
. When using a variable costing system, the contribution
margin (CM) discloses the excess of
a. Revenues over fixed costs.
b. Projected revenues over the breakeven point.
c. Revenues over variable costs.
d. Variable costs over fixed costs.
(Adapted)
14
. In an income statement prepared as an internal report
using the variable costing method, fixed factory
overhead would

a. Not be used.
b. Be used in the computation of operating income but
not in the computation of the contribution margin.
c. Be used in the computation of the contribution
margin.
d. Be treated the same as variable factory overhead.
(Adapted)
15
. Which of the following statements is true for a firm that
uses variable costing?

a. The cost of a unit of product changes because of


changes in number of units manufactured.
b. Profits fluctuate with sales.
c. An idle facility variation is calculated.
d. Product costs include direct (variable)
administrative costs.
(Adapted)

16
. Which method of inventory costing treats direct
manufacturing costs and manufacturing overhead
costs, both variable and fixed, as inventoriable costs?
a. Direct costing.

4
Multiple Choice Questions Variable vs. Absorption
Costing
b. Variable costing.
c. Absorption costing.
d. Conversion costing.
(Adapted)
17
. Using absorption costing, fixed manufacturing overhead costs
are best described as
a. Direct period costs.
b. Indirect period costs.
c. Direct product costs.
d. Indirect product costs.
18
. When a firm prepares financial reports by using
absorption costing,
a. Profits will always increase with increases in sales.
b. Profits will always decrease with decreases in
sales.
c. Profits may decrease with increase sales even if
there is no change in selling prices and costs.
d. Decreased output and constant sales result in
increased profit.
(Adapted)
19
. Which one of the following statements is correct
regarding absorption costing and variable costing?
a. Overhead costs are treated in the same manner
under both costing methods.
b. If finished goods inventory increases, absorption
costing results in higher income.
c. Variable manufacturing costs are lower under
variable costing.
d. Gross margins are the same under both costing
methods.
(Adapted)

20
. Absorption costing and variable costing are different
methods of assigning costs to units produced. Which
cost item listed below is not correctly accounted for as a
product costs?
Part of Product
Costs under
Absorption Variable
Cost Cost

5
Chapter 6 Variable vs.
Absorption Costing
a. Manufacturing supplies Yes Yes
b. Insurance on factory Yes No
c. Direct labor cost Yes Yes
d. Packaging and shipping Yes Yes

costs
(Adapted)

21
. The Blue Company has failed to reach its planned
activity level during its first 2 years of operation. The
following table shows the relationship among units
produced, sales, and normal activity for these years and
the projected relationship for year 3. All prices and costs
have remained the same for the last 2 years and are
expected to do so in year 3. Income has been positive
in both year1 and year 2.

Units Produced Sale


Planned Activity
Year 1 90,000 90,000
100,000
Year 2 95,000 95,000
100,000
Year 3 90,000 90,000
100,000

Because Blue Company uses an absorption costing


system, gross margin for year 3 should be

a. Greater than year 1.


b. Greater than year 2.
c. Equal to year 1.
d. Equal to year 2.
(Adapted)
22
. A company manufactures a single product for its
customers by contracting in advance of production.
Thus, the company produces only units that will be sold
by the end of each period. For the last period, the
following data were available:

Sales

6
Multiple Choice Questions Variable vs. Absorption
Costing

P40,000
Direct materials
9,050
Direct labor
6050
Rent (9/10 factory, 1/10 office0
3000
Depreciation on factory equipment
2000
Supervision (2/3 factory, 1/3 office)
1500
Salespeople’s salaries

1300
Insurance (2/3 factory, 1/3 office)

1200
Office supplies
750
Advertising
700
Depreciation on office equipment
500
Interest on loan

300

The gross margin percentage (rounded) was


a. 41%
b. 44%
c. 46%
d. 51%
23
. In an income statement prepared as an internal report,
total fixed costs normally are shown separately under

Absorption Costing Variable


Costing

a. No No
b. No Yes
c. Yes Yes
d. Yes No

7
Chapter 6 Variable vs.
Absorption Costing
(Adapted)
24
. A manufacturing company prepares income statements
using both absorption and variable costing methods. At
the end of a period, actual sales revenues, total gross
margin, and total contribution margin approximated
budgeted figures, whereas net income was substantially
below the budgeted amount. There were no beginning
or ending inventories. The most likely explanation of the
net income shortfall is that, compared to budget, actual

a. Sales price and variable costs had declined


proportionately.
b. Sales prices had declined proportionately more
than variable costs.
c. Manufacturing fixed costs had increased.
d. Selling and administrative fixed expenses had
increased.
(Adapted)
25
. Jansen, Inc. pays bonuses to its managers based on
operating income. The company uses absorption
costing, and overhead is applied on the basis of direct
labor hours. To increase bonuses, Jansen’s managers
may do all of the following except

a. Produce those products requiring the most direct


labor.
b. Defer expenses such as maintenance to a future
period.
c. Increase production schedules independent of
customer demands.
d. Decrease production of those items requiring the
most direct labor.
(Adapted)
26
. Net earnings determined using full absorption costing
can be reconciled to net earnings determined using
variable costing by computing the difference between

a. Inventoried fixed costs in the beginning and ending


inventories and any deferred over-or underapplied
fixed factory overhead.

8
Multiple Choice Questions Variable vs. Absorption
Costing
b. Inventoried discretionary costs in the beginning and
ending inventories.
c. Gross margin (absorption costing method) and
contribution
margin (variable costing method).
d. Sales as recorded under the variable costing method
and sales as recorded under the absorption costing
method.
(Adapted)

27
. The management of a company computes net income
using both the absorption and variable costing
approaches to product costing. In the current year, the
net income under the variable costing approach was
greater than the net income under the absorption
costing approach. His difference is most likely the result
of
a. A decrease in the variable marketing expenses.
b. An increase in the finished goods inventory.
c. An excess of sales volume over production volume.
d. Inflationary effects on overhead costs.
(Adapted)
28
. Net profit under absorption costing may differ from net
profit determined under variable costing. This difference
equals the change in the quantity of all units
a. An inventory times the relevant fixed costs per unit.
b. Produced times the relevant fixed costs per unit..
c. In inventory times the relevant variable cost per
unit.
d. Produced times the relevant variable cost per unit.
(Adapted)
29
. A company’s net income recently increased by 30%
while its inventory increase d to equal a full year’s sales
requirements. Which of the following accounting
methods would be most likely to produce the favorable
income results?
a. Absorption costing.
b. Direct costing
c. Variable costing.
d. Standard direct costing.

9
Chapter 6 Variable vs.
Absorption Costing
(Adapted)
30
. When comparing absorption costing with variable
costing, which of the following statements is true?
a. Absorption costing enables managers to increased
operating profits in the short run by increasing
inventories.
b. When sales volume is more than the production
volume, variable costing will result in higher
operating profit.
c. A manager who is evaluated based on variable
costing operating profit would be tempted to
increase production at the end of a period in order
to get a more favorable review.
d. Under absorption costing, operating profit is a
function of both sales volume and production
volume.
(Adapted)
31
. Fleet, Inc. manufactured 700 units of Product, a new
product, during the year. Product A’s variable and fixed
manufacturing costs per unit P6.00 and P2.00,
respectively. The inventory of Product A on December
31 consisted of 100 units. There was no inventory of
Product A on January 1. What would be the changed in
the peso amount of inventory on December 31 if
variable costing were used instead of absorption
costing?
a. P800 decrease.
b. P200 decrease.
c. P0
d. P200 increase.
(Adapted)
32
. During May, Roy Co. produced 10,000 units of Product
X. Costs incurred by Roy during May:

Direct materials

P10,000
Direct labor
20,000
Variable manufacturing overhead
5,000

10
Multiple Choice Questions Variable vs. Absorption
Costing
Variable selling and general expenses
3,000
Fixed manufacturing overhead
9,000
Fixed selling and general expenses
4,000
Total P
51,000

Under absorption costing, Product X’s unit cost was

a. P 5.10
b. P 4.40
c. P 3.80
d. P 3.50
(Adapted)
33
. During the month of April, Vane Co. produced and sold
10,000 units of a product. Manufacturing and selling
costs incurred during April were as follows:

Direct materials

P400,000
Variable manufacturing overhead
90,000
Fixed manufacturing overhead
20,000
Variable selling costs
10,000

The product’s unit cost under variable costing was

a. P 49
b. P 50
c. P 51
d. P 52
(Adapted)

Questions 34 and 35 are based on the following


information. Kirklin Co., a manufacturer operating at
95% of capacity, has been offered a new order at P7.25
per unit requiring 15% of capacity. No other use of the
5% current idle capacity can be found. However, if the

11
Chapter 6 Variable vs.
Absorption Costing
order were accepted, the subcontracting for the
required 10% additional capacity would cost P7.5 per
unit.

The variable cost of production for Kirklin on a per unit


basis follows:

Materials P3.50
Labor 1.50
Variable overhead 1.50
P6.50
34
. In applying the contribution margin approach to
evaluating whether to accept the new order, assuming
subcontracting, what is the average variable cost per
unit?
a. P6.83
b. P7.17
c. P7.25
d. P7.50
(Adapted)
35
. Assuming the average variable cost per unit of the new
order is P7.17, the expected contribution margin per
unit of the new order is
a. P.08
b. P.25
c. P.33
d. P.42
(Adapted)

Questions 36 and 37 are based on the following


information. A Company manufactures and sells single
product. Planned and actual production in 2002, its first
year of operation, was 100,000 units. Planned and
actual costs in 2002 were as follows:

Manufacturing Nonmanufacturing

Variable P600,000 P500,000

Fixed 400,000 300,000

12
Multiple Choice Questions Variable vs. Absorption
Costing
36
. Using absorption costing, the company’s operating
income in 20002 would be
a. P750,000
b. P900,000
c. P975,000
d. P1,020,000
(Adapted)
37
. Using variable costing, the company’s operating income
in 2002 would be
a. P750,000
b. P840,000
c. P915,000
d. P975,000
(Adapted)
38
. During its first year of operations, a company produced
275,000 units and sold 250,000 units. The following
costs were incurred during the year:

Variable costs per unit:


Direct materials P15.00
Direct labor 10.00
Manufacturing overhead 12.50
Selling and administrative 2.50
Total fixed costs:
Manufacturing overhead
P2,200,000
Selling and administrative 1,375,000

The difference between operating income calculated on


the absorption costing basis and on the variable costing
basis is that absorption-costing operating income is
a. P200,000 greater
b. P220,000 greater
c. P325,000 greater
d. P62,500 less
(Adapted)
39
. Keller Company, a manufacturer of rivets, uses
absorption costing. Keller’s manufacturing costs were
as follows:

Direct materials and direct labor


P800,000

13
Chapter 6 Variable vs.
Absorption Costing
Depreciation of machines
100,000
Rent of factory building
60,000
Electricity to run machines
35,000
How much of these costs should be inventoried?

a. P800,000
b. P835,000
c. P935,000
d. P995,000

Questions 40 through 43 are based on the following


information. Peterson Company’s records for the year
ended December 31 show that no finished goods
inventory existed at January 1 and no work in process
at the beginning or end of the year.

Net Sales

P1,400,000
Cost of goods manufactured: Variable P
630,000
Fixed P
315,000
Operating expenses: Variable P
98,000
Fixed P
140,000
Units manufactured
70,000
Units sold
60,000
40
. What is Peterson’s finished goods inventory cost at
December 31 under he variable costing method?
a. P90,000
b. P104,000
c. P105,000
d. P135,000
(Adapted)

14
Multiple Choice Questions Variable vs. Absorption
Costing
41
. What would be Peterson’s finished goods inventory cost
at December 31 under the absorption costing method?
a. P90,000
b. P104,000
c. P105,000
d. P135,000
(Adapted)
42
. Under the absorption costing method, Peterson’s
operating income for the year is
a. P217,000
b. P307,000
c. P352,000
d. P374,500
(Adapted)
43
. Under the variable costing method, Peterson’s
operating income for the year is
a. P217,000
b. P307,000
c. P352,000
d. P762,000
(Adapted)

Question 44 through 50 are based on the following


information. Valyn Corporation employs an absorption
costing system for internal reporting purposes; however,
the company is considering using variable costing. Data
regarding Valyn’s planned and actual operations for the
1998 calendar year are presented below.
Planned Actual
Activity Activity
Beginning finished goods
Inventory in units 35,000 35,000
Sales in units 140,000 125,000
Production in units 140,000 130,000

The planned per-unit cost figures shown in the schedule


were based on production and sale of 140,000 units in
2002. Valyn uses a predetermined manufacturing
overhead rate for applying manufacturing overhead to
its product; thus, a combined manufacturing overhead
rate of P9.00 per unit was employed for absorption
costing purposes in 2002. Any over-or-underapplied

15
Chapter 6 Variable vs.
Absorption Costing
manufacturing overhead is closed to cost of goods sold
at the end of the reporting year.

Planned Costs
Incurred
Per unit Total
Costs
Direct materials P 12.00 P1,680,000
P1,560,000
Direct labor 9.00 1,260,000
1,170,000
Variable manufac-
turing overhead 4.00 560,000
520,000
Fixed manufacturing
overhead 5.00 700,000
715,000
Variable selling
expenses 8.00 1,120,000
1,000,000
Fixed selling
expenses 7.00 980,000
980,000
Variable administrative
expenses 2.00 280,000
250,000
Fixed administrative
Expenses 3.00 420,000
425,000
Total P50.00 P7,000,000
P6,620,000

The 2002 beginning finished goods inventory for absorption


costing purposes was valued at the 2001 planned unit
manufacturing cost, which was the same as the 2002 planned
unit manufacturing cost. There are no work-in-process
inventories at either the beginning or the end of the year. The
planned and actual unit selling price for 2002 was P70.00 per
unit.
44
. The value of Valyn Corporation’s 2002 actual ending
finished goods inventory on the absorption costing basis
was
a. P900,000

16
Multiple Choice Questions Variable vs. Absorption
Costing
b. P1,200,000
c. P1,220,000
d. P2,000,000
(Adapted)
45
. The value of Valyn Corporation’s 2002 actual ending
finished goods inventory on the variable costing basis
was
a. P1,400,000
b. P1,200,000
c. P1,000,000
d. P750,000
(Adapted)
46
. Valyn Corporation’s actual manufacturing contribution
margin for 2002 calculated on the variable costing basis
was
a. P4,375,000
b. P5,500,000
c. P4,910,000
d. P5,625,000
(Adapted)
47
. Valyn Corporation’s total fixed costs in 2002 on the
absorption costing basis were

a. P2,095,000
b. P2,120,000
c. P2,055,000
d. P2,030,000
(Adapted)
48
. The total variable cost expensed in 2002 by Valyn
Corporation on the variable costing basis was
a. P4,375,000
b. P4,500,000
c. P4,325,000
d. P4,550,000
(Adapted)
49
. Valyn Corporation’s absorption costing operating
income in 2002 was
a. Higher than variable -costing operating income
because actual production exceeded actual sales

17
Chapter 6 Variable vs.
Absorption Costing
b. Lower than variable -costing operating income
because actual production exceeded actual sales
c. Lower than variable -costing operating income
because actual production was less than planned
production
d. Lower than variable -costing operating income
because actual sales were less than planned sales
(Adapted)
50
. The difference between Valyn Corporation’s 2002
operating income calculated on the absorption costing
basis and calculated on the variable costing basis was

a. P25,000
b. P45,000.
c. P75,000.
d. P100,000.
(Adapted)

Questions 51 through 54 are based on the following information.


The annual flexible budget below was prepared for use in
making decisions relating to Product X:

100,000 150,000
200,000
Units Units
Units
Sales volume P800,000 P1,200,000
P1,600,000

Manufacturing costs:
Variable P300,000 P 450,000 P
600,000
Fixed 200,000 200,000
200,000
P500,000 P 650,000 P
800,000

Selling and other


Expenses
Variable P200,000 P300,000
P400,000
Fixed 160,000 160,000
160,000

18
Multiple Choice Questions Variable vs. Absorption
Costing
P360,000 P460,000
P560,000
Income (or loss) P( 60,000) P 90,000
P240,000

The 200,000-unit budget has been adopted and will be used for
allocating fixed manufacturing costs to units of Product X. At the
end of the first 6 months, the following information is available:

Units
Production completed
120,000
Sales
60,000

All fixed costs are budgeted and incurred uniformly throughout


the year, and all costs incurred coincide with the budget. Over-
and underapplied fixed manufacturing costs are deferred until
year-end. Annual sales have the following seasonal pattern:
Portion of Annual
Sales
First quarter 10%
Second quarter 20%
Third quarter 30%
Fourth quarter 40%
51
. The amount of fixed factory costs applied to product
during the first six months under absorption costing is
a. Overapplied by P20,000
b. Equal to the fixed costs incurred.
c. Underapplied by P40,000.
d. Underapplied by P80,000.
(PhilCPA)
52
. Reported net income (or loss) for the first 6 months
under absorption costing is
a. P160,000
b. P0
c. P40,000
d. P(40,000)
(PhilCPA)

19
Chapter 6 Variable vs.
Absorption Costing
53
. Reported net income (or loss) for the first 6 months
under variable costing is

a. P180,000
b. P40,000.
c. P 0.
d. P (180,000)
(PhilCPA)
54
. Assuming that 90,000 units of Product X were sold
during the first 6 months and that this is to be used as a
basis, the revised budget estimate for the total number
of units to be sold during this year is
a. P360,000
b. P240,000
c. P200,000
d. P300,000
(PhilCPA)
Questions 55 and 56 are based on the following information:

The excerpt presented in the next column was taken


from Valenz Company’s records for the fiscal year ended
November 30.

Direct Materials used 300,000


Direct Labor 100,000
Variable Factory Overhead
50,000
Fixed Factory Overhead 80,000
Sell.. & admin. costs – variable 40,000
Sell.. & admin. costs –fixed
20,000
55
. If Valenz Company uses variable costing, the
inventoriable costs for the current fiscal year are

a. P400, 000.
b. P450, 000
c. P490, 000
d. P530, 000
(PhilCPA)
56
. Using absorption (full) costing, inventoriable costs are

20
Multiple Choice Questions Variable vs. Absorption
Costing
a. P400, 000
b. P450, 000
c. P530, 000
d. P590, 000
(PhilCPA)

Questions 57 and 58 are based on the following information:

Osawa, Inc. planned and actually manufactured 200,000 units


of its single product in 1998, its first year of operations. Variable
manufacturing costs were P30 per unit of product. Planned and
actual fixed manufacturing costs were P600, 000, and selling
and administrative costs totaled P400, 000 in 1998. Osawa sold
120,000 units of product in 1998 at a selling of P40 per unit.

57
. Osawa’s 1998 operating income using absorption (full)
costing is
a. P200, 000
b. P440, 000
c. P600, 000
d. P840, 000
(PhilCPA)
58
. Osawa’s 1998 operating income using variable costing
is
a. P200, 000
b. P440, 000
c. P800, 000
d. P600, 000
(PhilCPA)

Questions 59 and 60 are based on the following information:

A and B are autonomous segments of a corporation. They have


no beginning or ending inventories, and the number of units
produced is equal to the number of units sold. Following is
financial information relating to the two divisions:

A B
Sales P150,000 P400,000
Other revenue 10,000 15,000
Direct materials 30,000 65,000

21
Chapter 6 Variable vs.
Absorption Costing
Direct labor 20,000 40,000
Variable factory 5,000 15,000
overhead
Fixed factory overhead 25,000 55,000
Variable S&A expense 15,000 30,000
Fixed S&A expense 35,000 60,000
Central corporate 12,000 20,000
expenses (allocated)

59
. What is the total contribution to corporate profits
generated by Segment A before allocation of central
corporate expenses?

a. P18, 000
b. P20, 000
c. P30, 000
d. P90, 000
(PhilCPA)
60
. What is the contribution margin of Segment
B?

a. P150, 000
b. P205, 000
c. P235, 000
d. P265, 000
(PhilCPA)

Questions 73 and 74 are based on the following information:

Herman, Marco, and Alexis formed a partnership on January 1,


2004, and contributed P150,000, P200,000, and P250,000,
respectively. Their articles of co-partnership provide that the
operating income be shared among the partners as follows: as
salary, P24,000 for Herman, P18,000 for Marco, and P12,000
for Alexis; interest of 12% on the weighted average capital
during 2004 of the three partners; and, the remainder in the
ratio of 2:4:4, respectively. The operating income for the year
ending December 31, 2004 amounted to P176,000. Herman
contributed additional capital of P30,000 on July 1 and made a
drawing of P10,000 on October 1; Marco contributed additional
capital of P20,000 on August 1 and made a drawing of P10,000

22
Multiple Choice Questions Variable vs. Absorption
Costing
on October 1; and, Alexis made a drawing of P30,000 on
November 1.
61
. The division of the P176,000 operating income is:
a. Herman, P53,760; Marco, P62,520; and, Alexis,
P59,720
b. Herman, P35,200; Marco, P70,400; and, Alexis,
P70,400
c. Herman, P48,400; Marco, P66,800; and, Alexis,
P60,800
d. Herman, P53,180; Marco, P62,060; and Alexis,
P60,760
(PhilCPA)
62
. The partners’ capital balances on December 31, 2004
are:
a. Herman, P179,680; Marco, P229,360; and, Alex,
P239,360
b. Herman, P179,760; Marco, P229,520; and, Alex,
P239,520
c. Herman, P189,680; Marco, P239,360; and, Alex,
P269,360
d. Herman, P223,180; Marco, P272,060; and, Alex,
P280,760
(PhilCPA)
63
. Mr. Zoom and his very close friend, Mr. Boom, formed a
partnership on January 1, 2004, with Zoom contributing
P16,000 in cash and Boom contributing equipment, with
book value of P6,400 and fair value of P4,800, and
inventory items, with book value of P2,400 and fair
value of P3,200. During 2004, Boom made additional
investments of P1,600 on April1 and P1,600 on June 1,
and withdrew P4,000 on September 1. Zoom had no
additional investments or withdrawals during the year.
What was the average capital balance of Mr. Boom
during 2004?
a. P9,600.
b. P8,800.
c. P8,000.
d. P7,200.
(PhilCPA)
64
. On January 1, 2004, Zeep and Beep have capital
balances of P20,000 and P16,000, respectively. On

23
Chapter 6 Variable vs.
Absorption Costing
July 1, 2004, Zeep invested an additional P4,000 while
Beep withdrew P1,000. Profits and losses are divided
as follows: Beep is the managing partner and as such
shall receive P16,000 as salary, with Zeep receiving
P7,200; both partners shall receive interest of 10% on
their beginning capital balances, to offset whatever
difference in capital investments they have; and, any
remainder shall be divided equally. The net income of
the partnership for 2004 was P9,600.
What was Zeep’s share in the net income for 2004?
a. P9,200.
b. P4,800.
c. P 880.
d. P 600.
(PhilCPA)
65
. Moonbits Partnership had a net income of P8,000 for
the month ended September 30, 2004. Sunshine
purchased an interest in Moonbits Partnership of Liz
and Dick by paying Liz P32,000 for one-half of her
capital and one-half of her 50% profit-sharing interest
on October 1, 2004. At this time Liz’s capital balance
was P24,000 and Dick’s capital balance was P56,000.

Sunshine should receive a credit to her capital account


of:
a. P12,000.
b. P16,000.
c. P20,000.
d. P26,667.
(PhilCPA)
66
. Dulce Martin, a partner in a partnership that carries the
name of the Sweet Shop, has a 30% participation in
partnership profits. Her capital account had a net
decrease of P48,000 during 2004. In the same year,
she withdrew P104,000 (charged against her capital
account) and contributed property valued at P20,000 to
the partnership.

The net income of the partnership for 2004 was:


a. P36,000.
b. P120,000.
c. P132,000.

24
Multiple Choice Questions Variable vs. Absorption
Costing
d. P440,000.
(PhilCPA)
67
. Michelle, an active partner in the Michelle-Esme
Partnership, receives an annual bonus of 25% of the
partnership income after deducting the bonus. For the
year ended December 31, 2004, the partnership income
before the bonus amounted to P240,000.

The bonus of Michelle for the year 2004 is:


a. P45,000.
b. P48,000.
c. P60,000.
d. P80,000.
(PhilCPA)

From the following data from the records of ABACA partnership,


answer Questions 80 through 82.

ABACA Partnership
Balance Sheet
December 31, 2002

Assets
Cash P
2,000
Other non-cash assets
28,000
Total assets P
30,000

Liabilities & Net Worth


Liabilities P
5,000
Aca, loan
2,500
Aca, capital
12,500
Baca, capital
7,000

25
Chapter 6 Variable vs.
Absorption Costing
Caca, capital
3,000
Total liabilities and capital P
30,000

Profit and loss ratio is 3:2:1 for Aca, Baca, and Caca,
respectively. The other non-cash assets were realized as
follows:

Date Cash Received Book


Value
January 2003 P6,000
P9,000
February 2003 3,500
7,700
March 2003 12,500
11,300

Cash is distributed as assets are realized.


68
. The total loss to Aca is:
a. P3,000.
b. P2,000.
c. P1,000.
d. P-0-
(PhilCPA)
69
. Total cash received by Baca is:
a. P2,200.
b. P-0-.
c. P5,000.
d. P1,500.
(PhilCPA)
70
. The amount of cash received by Caca in January is
equal to:
a. P200.
b. P1,000.
c. P500.
d. P-0-
(PhilCPA)

26
Multiple Choice Questions Variable vs. Absorption
Costing
71
. The admission of a new partner to a 20% interest in a
partnership for an investment of P18,000, with total
agreed capital to be P75,000, will result in:
a. Goodwill to the old partners.
b. Goodwill to the new partner.
c. Bonus to the old partners.
d. Bonus to the new partner.
(PhilCPA)
72
. X, Y, and Z have capital balances of P40,000, P50,000,
and P18,000 and a profit-sharing ratio of 4:2:1,
respectively. If X received P8,000 upon liquidation of
the partnership, the total amount received by all the
partners was:
a. P108,000.
b. P56,000.
c. P52,000.
d. P24,000.
(PhilCPA)
73
. Assume the same facts in the preceding question,
except that X received P26,000 as a result of the
liquidation. Z received, as part of the liquidation, the
amount of:
a. P26,000.
b. P14,500.
c. P18,000.
d. P14,000.
(PhilCPA)
74
. JC, a partner in the JC Partnership, has a 30%
participation in the partnership profit and loss. His
capital account had a net decrease of P60,000 in 2004.
In the same year, he withdrew P130,000 from the
partnership against his capital and invested property
valued at P25,000, in the partnership. The net income
of the partnership in 2004 is:
a. P150,000.
b. P233,333.
c. P350,000.
d. P550,000
(PhilCPA)
Questions 87 through 91 are based on the following
information:

27
Chapter 6 Variable vs.
Absorption Costing
On May 1, 2003, the business assets of John and Paul were as
summarized below:
John
Paul
Cash P 11,000 P
22,354
Accounts receivable 234,536
567,890
Inventories 120,035
260,102
Land 603,000
-
Building -
428,267
Furniture and fixtures 50,345
34,789
Other assets 2,000
3,600
Total P1,020,916
P1,317,002
Accounts payable P 178,940 P
243,650
Notes payable 200,000
345,000
John, capital 641,976
-
Paul, capital -
728,352
Total P1,020,916
P1,317,002
75
. The capital accounts of the partners after adjustment
will be:
a. John, P614,476; Paul, P683,052.
b. John, P615,942; Paul, P717,894.
c. John, P640,876; Paul, P712,345.
d. John, P613,576; Paul, P683,350.
(PhilCPA)

76
. How much assets does the partnership have?
a. P2,337,918.
b. P2,237,918.
c. P2,265,118.
d. P2,365,218.

28
Multiple Choice Questions Variable vs. Absorption
Costing
(PhilCPA)
77
. Assuming Peter offered to join for a 20% interest in the
firm. How much cash should he contribute?
a. P330,870.
b. P337,487.
c. P344,237.
d. P324,382.
(PhilCPA)
78
. After Peter’s admission, the profit and loss sharing ratio
was agreed to be 4:4:2 based on capital credits. How
much should the cash settlement be between John and
Paul?
a. P33,602.
b. P32,930.
c. P32,272.
d. P34,288.
(PhilCPA)
79
. During the first year of their operations, the partners
earned P325,000. Profits were distributed in the agreed
manner. The partners’ drawings were: John, P50,000;
Paul, P65,000; and, Peter, P28,000. How much are the
partners’ capital balances after the first year?
a. John, P750,627; Paul, P735,177; Peter, P372,223.
b. John, P728,764; Paul, P713,764; Peter, P361,382.
c. John, P757,915; Paul, P742,315; Peter, P375,837.
d. John, P743,121; Paul, P727,825; Peter, P368,501.
(PhilCPA)
80
. Bel, Mo, and Ko, new CPAs, are to form a partnership.
Bel will contribute cash of P50,000 and his computer
that originally cost him P60,000 but with a second hand
value of P25,000. Mo will contribute P80,000 in cash.
Ko, whose family sells computers, will contribute
P25,000 in cash and a brand-new computer with printer
that cost his family’s computer dealership P50,000 but
with a regular selling price of P60,000. The three
agreed to share profits and losses equally. Upon
formation, capital balances are:
a. Bel, P75,000; Mo, P80,000; and, Ko, P85,000.
b. Bel, P80,000; Mo, P80,000; and, Ko, P80,000.
c. Bel, P88,333; Mo, P88,333; and, Ko, P88,334.
d. Bel, P110,000; Joy, P80,000; and, Ko, P75,000.
(PhilCPA)

29
Chapter 6 Variable vs.
Absorption Costing
81
. Black and White are partners who have capital
balances of P600,000 and P480,000, and sharing
profits in the ratio of 3:2. Blue is admitted as a partner
upon investing P220,000 for a 25% interest in the firm,
and profits are to be shared equally. Given the choice
between goodwill and bonus methods, Blue would:
a. Prefer bonus method due to Blue’s gain of
P105,000.
b. Prefer bonus method due to Blue’s gain of
P140,000.
c. Prefer goodwill method due to Blue’s gain of
P140,000.
d. Be indifferent for goodwill and bonus methods are
the same.
(PhilCPA)
82
. Mata, Tali, and Lino are partners with capital credit
balances as of December 31, 2004 of P300,000,
P300,000 and P200,000, respectively. Lino is allowed
to withdraw, and it is agreed that he is to take certain
furniture items at their second-hand value of P12,000,
plus a promissory note for the balance of his interest.
The furniture items are carried on the books as fully
depreciated; brand new, however, they would cost
P20,000. If profits and losses are shared equally, the
acquisition of the furniture items by Lino would result in:
a. Increase in capital of P4,000 each for Mata, Tali,
and Lino.
b. Decrease in capital of P6,000 each for Mata, Tali,
and Lino.
c. Increase in capital of P8,000 for Lino.
d. Decrease in capital of P8,000 for Lino
(PhilCPA)
83
. Dino and Gavino, partners, allow monthly salaries
(P6,000 & P5,000, respectively) and 6% interest on
beginning capital (P300,000 & P230,000, respectively),
and then divide any remaining profit equally.
On a net profit of P100,000, the respective shares
would be:
a. P50,000 and P50,000.
b. P54,500 and P45,500.
c. P56,600 and P43,400.
d. P58,100 and P41,900.

30
Multiple Choice Questions Variable vs. Absorption
Costing
(PhilCPA)
84
. Jon, Lee, and Vi are partners sharing profits 30:20:50,
respectively. The partners agreed to dissolve their
partnership and, upon liquidation, all of the partnership’s
non-cash assets are sold and sufficient cash is realized
to pay all claims except one for P50,000. Vi is
personally insolvent, but the other two partners are
capable of meeting any indebtedness of the firm. Of the
remaining claim against the firm, Jon is to absorb:
a. P15,000.
b. P25,000.
c. P30,000.
d. P40,000.
(PhilCPA)
85
. Marco and Tonio are partners with capital balances of
P200,000 and P100,000, and sharing profits and losses
in the ratio of 3:1, respectively. They agree to admit a
new partner, Claire. Claire invests P125,000 for a 25%
interest in the firm and the parties agree that the total
firm capital after Claire’s admission is to be P425,000.
After Claire’s admission, the partners’ capital balances
would be:
a. P214,062.50, P104,687.50, and P106,250,
respectively.
b. P225,000.00, P100,000.00, and P100,000,
respectively.
c. P239,062.50, P79,687.50, and P125,000.00,
respectively.
d. P250,000.00, P75,000.00, and P100,000.00,
respectively.
(PhilCPA)
86
. The condensed balance sheet of the partnership of Ken
Sy and Ben Ty as of December 31, 2004 showed the
following:

Total assets P
200,000
Total liabilities
40,000
Ken Sy, capital
80,000
Ben Ty, capital
80,000

31
Chapter 6 Variable vs.
Absorption Costing

On this date, the partnership was dissolved and its net


assets were transferred to a newly formed corporation.
The fair value of the assets was P24,000 more than the
carrying value on the firm’s books. Each of the partners
was issued 10,000 shares of the corporation’s P1 par
common stock. Immediately after effecting the transfer
of the net assets, and the issuance of stock, the
corporation’s additional paid-in capital account would be
credited for:
a. P136,000.
b. P140,000.
c. P154,000.
d. P164,000.
(PhilCPA)
87
. Red, White and Blue are partners who had average
capital balances, respectively, of P240,000, P120,000,
and P80,000 during 2004. Partners receive 10%
interest on their average capital balances. After
deducting salaries of P60,000 for Red and P40,000 for
Blue, the residual profit or loss is divided equally. In
2004, the partnership sustained a P66,000 loss before
the interest and salaries to partners.
By what amount would the capital account of Red
change?
a. P14,000 increase.
b. P22,000 increase.
c. P70,000 decrease.
d. P84,000 decrease.
(PhilCPA)
88
. Alma, Bona, and Cora are to form a partnership. Alma
is to contribute cash of P100,000; Bona, P10,000, and
Cora, P100,000. Alma and Cora are not to actively
participate in the business but will refer customers,
while Bona will manage the firm. Bona has to give up
her present job which gives her an annual income of
P120,000. The partners decided that profits and losses
shall be shared equally. Upon formation, partner’s
capital balances would be:
a. P70,000, P70,000, and P70,000, respectively.
b. P100,000, P10,00, and P100,000 respectively.
c. P100,000, P130,000, and P100,000, respectively.

32
Multiple Choice Questions Variable vs. Absorption
Costing
d. P110,000, P110,000, and P110,000, respectively.
(PhilCPA)
89
. Partners Rob and Roy, who share equally in profits and
loses, have the following balance sheet as of December
31, 1999:
Cash P120,000 A/Payable
P172,000
A/receivable 100,000 Accum. Dep’n.
8,000
M/Inventory 140,000 Rob, capital
140,000
Equipment 80,000 Roy, capital
120,000
Total P440,000 Total equities
P440,000
They agreed to incorporate their partnership, with the
new corporation absorbing the net assets after the
following adjustments: provision of allowance for bad
debts of P10,000; statement of the inventory at its
current fair value of P160,000; and, recognition of
further depreciation on the equipment of P3,000. The
corporation’s capital stock is to have a par value of
P100, and the partners are to be issued corresponding
total shares equivalent to their adjusted capital
balances.
The total par value of the shares of capital stock that
were issued to partners Rob and Roy was:
a. P260,000
b. P267,000
c. P273,000
d. P280,000
(PhilCPA)
90
. The partnership agreement of Mel and Jay provides that
“interest of 10% per annum is to be credited to each
partner on the basis of weighted average capital
balances”. A summary of Jay’s capital account for the
year just ended follows:
January 1 balance
P280,000
July 1 additional investment
80,000

33
Chapter 6 Variable vs.
Absorption Costing
August 1 capital withdrawal
30,000
December 31 balance 330,000
For the year just ended, the amount of “interest” that
was credited to Jay was:
a. P30,500
b. P30,750
c. P33,000
d. P34,500
(PhilCPA)
91
. TVJ Partnership provided for the following in their
distribution of profits and losses:
First, T is to receive 10% of net income up to
P100,000 and 20% of the amount in excess thereof;
Then, V and J are each t o receive 5% of the
remaining income in excess of P150,000 after T’s share
above;
Finally, the balance is to be distributed equally to
the three partners.
If the partnership earned a net income of P250,000, the
total share of partner T would be:
a. P100,000
b. P108,000
c. P110,000
d. P130,000
(PhilCPA)
92
. Partners Ivy, Jay, and Kay, who divide profits 4:3:3,
have the following condensed balance sheet:
Assets P1,880,000 Liabilities P
480,000
Ivy, capital
620,000
Jay, capital
400,000
Kay, capital
380,000
Total P1,880,000 Total P
1,880,000
Lee will be admitted as a new partner with a 20%
interest, after he pays the three partners a premium of
10%. Lee’s capital credit will b:
a. P200,000

34
Multiple Choice Questions Variable vs. Absorption
Costing
b. P280,000
c. P350,000
d. P376,000
(PhilCPA)
93
. Mac and Nat are partners with capitals of P200,000 and
P100,000 and sharing profits and losses at 3:1,
respectively. They decided to admit Odi as a new
partner with a 50% interest in the firm. Odi invested
cash of P150,000, and Mac and Nat transferred
portions of their capitals as a bonus to Odi. After
admission, Nat’s capital would be:
a. P37,500
b. P56,250
c. P81,250
d. P100,000
(PhilCPA)
94
. Paco, Quin, and Romy are partners with capital
balances on June 30, 19-9 of P300,000, P300,000, and
P200,000, respectively, and sharing profits and losses
equally. Romy is to retire, and it is agreed that he is to
take certain furniture (with second-hand value of
P50,000) and a note for his interest. The furniture is
carried in the books at P65,000, but brand new would
cost P80,000. Romy’s acquisition of the furniture would
result in:
a. Reduction in capital of P5,000 each for Paco, Quin,
and Romy.
b. Reduction in capital of P7,500 each for Paco, and
Quin.
c. Reduction in capital of P15,000 for Romy.
d. Reduction in capital of P55,000 for Romy.
(PhilCPA)
95
. Bach, Lizst, and Strauss, sharing profits and losses
4:4:2, decided to liquidate their partnership. Just prior
to liquidation, the partnership’s condensed balance
sheet was as follows:
Cash P100,000 Liabilities
P140,000
Other Assets 400,000 Bach, loan
10,000

35
Chapter 6 Variable vs.
Absorption Costing
Bach, capital
45,000
Lizst, capital
105,000
___ _____ Strauss, capital
200,000
Total P500,000 Total
P 500,000
The other assets were sold for P247,500, and the
partners agreed to make additional cash contributions
to answer for any capital deficiency. Identify the
deficient partner, and indicate his additional cash
contribution to finally liquidate the partnership:
a. Bach, P6,000.
b. Bach, P16,000.
c. Lizst, P30,500.
d. Strauss, P44,000.
(PhilCPA)
96
. Sanchez and Tan are partners sharing profits equally
and with capital balances, respectively, of P750,000 and
P500,000. The firm owes Tan P200,000, as evidenced
by a promissory note. Upon liquidation, cash of
P300,000 becomes available for distribution to the
partners. In the final cash distribution, the respective
shares of Sanchez and Tan will be:
a. P150,000 and P150,000
b. P175,000 and P125,000
c. P200,000 and P100,000
d. P275,000 and P 25,000
(PhilCPA)
97
. Anna, Carla, and Ella are partners sharing equally in
profits, after allowing a bonus to the managing partner.
Anna is the managing partner, and she is entitled to a
bonus of 20% of the profit after bonus. If the
partnership realized a net income of P360,000 in 2004,
Anna’s bonus was:
a. P60,000.
b. P72,000.
c. P80,000.
d. P96,000.
(PhilCPA)

36
Multiple Choice Questions Variable vs. Absorption
Costing
98
. Partners Jose, Luciano, and Placido have average
capital balances of P240,000, P120,000, and P80,000,
respectively, during 2004. Each partner receives 10%
interest on his average capital balance. After deducting
salaries of P60,000 for Jose and P40,000 for Placido,
the residual profit or loss is divided equally. In 2004, the
partnership sustained a P66,00 loss before partners’
interests and salaries. By how much would Placido’s
capital account change?
a. P20,000 increase.
b. P22,000 decrease.
c. P32,000 decrease.
d. P48,000 increase.
(PhilCPA)
99
. Partners Nora and Vilma divide profits and losses in the
respective ratio of 3:2. On December 31, 2005, their
capital accounts had balances of P120,000 for Nora
and P80,000 for Vilma. On this date, Lorna was
admitted as a new partner with a one-third interest in
capital and profits for an investment of P80,000. The
capital of the new partnership was to be P300,000.
Immediately after the admission of the new partner,
Nora’s capital was:
a. P108,000.
b. P112,000.
c. P120,000.
d. P132,000.
(PhilCPA)
100
. The following is the condensed balance sheet of the
partnership, as of December 31, 2005, of Athos,
Porthos, and Aramis who share profits in a 4:3:3 ratio.

Cash P 180,000 Accts. Payable P


420,000
Athos, receivable 40,000 Aramis, loan
60,000
Other assets 1,660,000 Athos, capital
620,000
Porthos, capital
400,000

37
Chapter 6 Variable vs.
Absorption Costing
________ Aramis, capital
380,000
Total P1,880,000 Total equities P
1,880,000
Assume that the assets and liabilities of the partnership
are fairly valued and that the partners decided to admit
D’Artagnan as a partner with a 20% interest, but with
neither bonus nor goodwill to be recorded, how much
should D’Artagnan contribute in cash or other assets?
a. P280,000.
b. P284,000.
c. P350,000.
d. P355,000.
101
. Luz, Vi, and Minda are partners with capital balances,
as of December 31, 2004, of P300,000, P300,000 and
P200,000, respectively, and who share profits and
losses equally. Minda wishes to withdraw, and it is
agreed that she is to take certain furniture items, with
second hand value of P50,000 and a note for the
balance of her interest. The furniture items are carried
in the books at P65,000; brand new, however, they
would cost P80,000. the value of the note that Minda
would get is:
a. P120,000.
b. P135,000.
c. P145,000.
d. P150,000.
102
. Juan, Pedro, and Pablo are partners who share profits
and losses in a 5:3:2 ratio and, on January 1, 2004,
have capital balances of P90,000, P160,000, and
P200,000, respectively. Pablo withdrew from the
partnership on July 1, 2004 and the partners agreed
that, as of this date, certain inventory items would have
to be revalued at P70,000 from their recorded cost of
P50,000. For the six-month period ending June 30,
2004, the partnership realized a net income of
P130,000. The partners decided that Pablo should be
paid P245,000 for his interest and the remaining
partners’ capital accounts should be adjusted for any
goodwill resulting from the settlement. The payment to
Pablo included goodwill of:
a. P15,000.

38
Multiple Choice Questions Variable vs. Absorption
Costing
b. P25,000.
c. P42,500.
d. P50,000.
103
. The condensed balance sheet of the partnership of Tic,
Tac, and Toe as of June 30, 2004, with their
corresponding profit and loss share, follows:

Net assets

P400,000

Tic, capital (50%)

P200,000
Tac, capital (30%)
120,000
Toe, capital (20%)
80,000
Total capital

P400,000

As of said date, Tic retired from the partnership. Per


agreement, Tic was paid P225,000 for his interest and
the goodwill implied from the settlement was recorded.
After Tic’s retirement, the partnership’s “net assets”
was:
a. P175,000.
b. P200,000.
c. P225,000.
d. P250,000.
(PhilCPA)
104
. After operating for five years, the partnership of Remy
and Martin, who share profits and losses equally, had
balances as follows:

Net assets
P130,000
Remy, capital
85,000
Martin, capital
45,000

39
Chapter 6 Variable vs.
Absorption Costing

If liquidation takes place at this time and the assets are


realized at book value, Remy and Martin would be
entitled to receive:
a. P65,000 each.
b. P85,000 and P45,000, respectively.
c. P90,000 and P40,000, respectively.
d. P97,500 and P32,500, respectively.
(PhilCPA)
105
. The partners Alma, Bida, Cita, and Dina, who share
profits and losses in the respective ratio of 3:3:2:2,
decided to liquidate their partnership. Just prior to
liquidation, they prepared the following summary
balance sheet:

Cash P 100,000 Liabilities


P 750,000
Other assets 1,800,000 Bida, loan
160,000
Dina, loan
50,000
Alma, capital
420,000
Bida, capital
215,000
Cita, capital
205,000
__________ Dina, capital
100,000
Total P1,900,000 Total
P1,900,000
The noncash assets realized P800,000. If all the
partners are personally solvent, deficiency/deficiencies
resulting from the liquidation process, will require
additional cash from:
a. Bida at P85,000 and Dina at P100,000.
b. Bida at P85,000.
c. Dina at P50,000.
d. Bida at P50,000.

40
Multiple Choice Questions Variable vs. Absorption
Costing
(PhilCPA)
106
. Mac, Kuh, and Nat, partners sharing profits and losses
equally, decided to form a corporation. They have
capital balances, respectively, of P100,000, P100,000,
and P200,000, and all of their assets and liabilities will
be transferred to the corporation. Their net assets will
be revalued from P400,000 to P550,000, with the
substantial revaluation due to land which was originally
contributed by Nat at P100,000. At P10 par value, the
partners are to receive shares of stock as follows:
a. 10,000, 10,000, and 35,000, respectively.
b. 12,500, 12,500, and 30,000, respectively.
c. 15,000, 15,000, and 25,000, respectively.
d. 18,333, 18,333, and 18,334, respectively.
(PhilCPA)
107
. Allan and Alex formed a partnership and they agreed to
share initial capital equally, although Allan contributed
P150,000 and Alex contributed P126,000 in identifiable
assets. Under the bonus approach to adjust the capital
accounts, Alex’s unidentifiable assets should be debited
for:
a. P-0-
b. P12,000.
c. P24,000.
d. P69,000.
(PhilCPA)
108
. On October 1, 2005, Carla and Clara joined in a
partnership. Carla contributed cash while Clara
contributed merchandise worth P25,000 and a one-year
chattel mortgage note for P15,000. If initial capital
balances are to conform to the profit-sharing ratio of
2:3, respectively, the amount of cash contributed by
Carla was:
a. P24,000.
b. P30,000.
c. P40,000.
d. P50,000.
(PhilCPA)
109
. The partnership agreement of Bing and Bong provides
that Bing is to receive a 20% bonus on profits before the

41
Chapter 6 Variable vs.
Absorption Costing
bonus. Remaining profits and losses are divided in the
respective ratio of 2:3. Which partner has a greater
advantage when the partnership realizes a profit or
when it sustains a loss?
Profit Loss
a. Bing Bong
b. Bing Bing
c. Bong Bing
d. Bong Bong
(PhilCPA)
110
. Digno and Dindo, who share partnership profits and
losses according to their weighted average capital ratio,
had capital accounts in 2005 as follows:

Digno, Capital
Dindo,Capital

4/1 5,000 1/1 48,000 9/1 3,000 1/1


25,000
7/31 10,000 10/1 1,000 6/1
10,000
9/30 5,000 11/30
5,000
12/1 4,000

If the partnership reports a net income of P60,000 in


2005, Digno’s share would be:
a. P20,000.
b. P22,500.
c. P37,500.
d. P40,000.
(PhilCPA)

111
. Tino and Tito, sharing profits and losses in a 2:3 ratio,
have respective capitals of P125,000 and P175,000.
They agreed to admit Tony as a new partner with a one-
third interest in both capital and profits, for an
investment of P200,000, after re-valuing the assets of
Tino and Tito. Goodwill to the original partners would
be:
a. P-0-

42
Multiple Choice Questions Variable vs. Absorption
Costing
b. P66,667.
c. P100,000.
d. P133,333.
(PhilCPA)

Questions 124 and 125 are based on the following:


112
. Vida, Vina, and Vita, sharing profits and losses 50%,
30%, and 20%, have capital credit balances of P40,000,
P30,000, and P20,000, respectively. They decided to
admit a new partner, Vera, to a 30% interest in the
partnership upon Vera’s investment of an amount equal
to five-sixths of her capital credit with no asset
adjustment recognized. Immediately after the
admission of Vera, the capital credit balance of Vina will
be:
a. P28,200.
b. P30,000.
c. P31,800.
d. P33,000.
(PhilCPA)
113
. Assuming that instead of admitting Vera, it was mutually
agreed that Vita should retire from the partnership and
she would be paid P24,000 in full settlement of her
interest but with no asset adjustment is to be
recognized. Immediately after the retirement of Vita,
the capital credit balance of Vida will be:
a. P32,000.
b. P36,000.
c. P37,500.
d. P40,000.
(PhilCPA)
114
. Gardan and Gordon formed a partnership on July 1,
2005 to operate two stores to be managed by each of
them. They invested P30,000 and P20,000 and agreed
to share earnings 60% and 40%, respectively. All their
transactions were for cash, and all their subsequent
transactions were handled through their respective bank
accounts, summarized as follows:
Gardan
Gordon

43
Chapter 6 Variable vs.
Absorption Costing
Cash receipts P79,100
P65,245
Cash disbursements 62,275
70,695

On October 31, 2005, all remaining noncash assets in


the two stores were sold for cash of P60,000. The
partnership was dissolved, and cash settlement was
effected. In the distribution of the P60,000 cash,
Gardan received:
a. P24,000.
b. P26,000.
c. P34,000.
d. P36,000.
(PhilCPA)
115
. On October 31, 2005, Ivy, Irma, and Irene, who share
earnings 5:3:2 respectively, decided to liquidate their
partnership at which time their condensed balance
sheet was as follows:

Cash P 50,000 Liabilities P


60,000
Other assets 250,000 Ivy, capital
80,000
Irma, capital
90,000
________ Irene, capital
70,000
Total assets P 300,000 Total equities P
300,000

If the first cash sale of assets booked at P150,000


resulted in net realization of P120,000, the amount to be
distributed to Irene would be:
a. P15,000.
b. P44,000.
c. P51,000.
d. P60,000.
(PhilCPA)
116
. On May 31, 2006, Aldo, Alex and Andy joined in a
business partnership. Aldo invested cash of P25,000.
Alex invested equipment with cost of P35,000, book

44
Multiple Choice Questions Variable vs. Absorption
Costing
value of P20,000, and fair value of P27,500. Andy
invested property with initial cost of P25,000, carrying
value of P20,000, and market value of P40,000, and the
partnership accepted responsibility for the mortgage of
P17,500 attached to the property. The partnership
agreement provides that profits and losses shall be
divided equally, but is silent regarding capital
contributions. The partner with the largest May 31,
2006 capital account balance is:

a. Aldo.
b. Alex.
c. Andy.
d. All-equally.
(PhilCPA)
117
. Chona and Charo formed a partnership on May 31,
2006. Chona’s contribution consisted of her
proprietorship’s net assets with current fair value of
P60,000. Charo contributed enough cash to secure a
one-fourth interest in the partnership. If Chona is
allowed goodwill credit equal to 20% of her capital,
Charo’s cash contribution was:
a. P15,000.
b. P20,000.
c. P25,000.
d. P30,000.
118
. Bart and Bert, on January 1, 2006, have respective
capital balances of P30,000 and P50,000. Bart
invested an additional P30,000 on June 30, while Bert
invested an additional P60,000 on November 1. Bart
had a capital withdrawal of P12,000 on August 1, while
Bert had a capital withdrawal of P20,000 on September
30. The partners allow respective monthly salaries of
P500 and P600, credit 15% interest on average capital
balances, and share any residual earnings equally. If
the partnership reports an operating income of P25,000
for 2006, Bart’s distributive share would be:
a. P10,775.
b. P12,500.
c. P14,225.
d. P15,000.

45
Chapter 6 Variable vs.
Absorption Costing
119
. Dahlia, a partner of the 3-D Co., has a twenty-five
percent participation in the partnership’s operating
results. During the year 2006, Dahlia transferred
equipment with fair value of P25,000 to the partnership
but she made a regular weekly cash drawings of P250
for personal use. On December 31, 2006, Dahlia’s
capital credit balance is increased by P27,000. What
was the partnership’s net income (loss) in 2006?
a. P(15,000).
b. P48,000.
c. P56,000.
d. P60,000.

Questions 132 and 133 are based on the following information:

Terry and Timmy entered into a partnership on May 31, 2006,


contributing cash of P48,000 and P32,000, respectively, and
agreeing to divide earnings in the ratio of their initial investments
after allowing annual salary allowance of P12,000 each. On
December 31, 2006, the Income Summary account had a credit
balance of P34,000, while the drawing accounts showed debit
balances of P14,000 for Terry and P10,000 for Timmy.

At the beginning of the next year, Tommy was admitted into the
firm as a new partner with a 33-1/3% interest for a capital credit
equal to his cash investment of P60,000. Terry and Timmy then
effected a private cash settlement between themselves in order
to make the capital balances conform to a new profit-sharing
ratio of 4:2:3, respectively, with salary allowances scrapped.
120
. How much was the amount of goodwill, if any, that was
recognized in connection with the admission of the new
partner?
a. P20,000.
b. P24,000.
c. P30,000.
d. P36,000.
(PhilCPA)
121
. How much was the amount of the private cash
settlement effected between the old partners?
a. P5,000.
b. P9,000.
c. P12,000.

46
Multiple Choice Questions Variable vs. Absorption
Costing
d. P15,000.
(PhilCPA)

Questions 134 and 135 are based on the following information:

Eli, Emi, and Epi divide profits and losses in a 2:3:4 ratio. Just
prior to liquidating their partnership, their respective capital
account balances were P50,000, P96,000, and P74,000 as of
April 1, 2006. Their total assets include cash of P5,000 and a
loan to Eli for P10,000, while their total liabilities of P90,000
include a loan from Epi for P30,000. The partners agreed to
distribute cash, as it becomes available, at each month-end.
Realization proceeds were P68,000 in April, P56,000 in May,
and P63,000 in June.
122
. In the cash distribution on May 31, 2006, the distributive
share of Emi amounted to:
a. P-0-
b. P13,000.
c. P26,000.
d. P39,000.
(PhilCPA)
123
. In the cash distribution on June 30, 2006, the
distributive share of Epi amounted to:
a. P14,000.
b. P21,000.
c. P28,000.
d. P35,000.
(PhilCPA)
124
. On October 1, 2006, Al and Bino pooled their resources
in a partnership, with the firm taking over their business
assets and assuming their business liabilities. The
partners’ capitals are to be based on net assets
transferred, after these adjustments: Bino’s inventory is
to be increased by P3,000; allowance for bad debts for
P1,000 and P1,500 are to be set up, respectively, in the
books of Al and Bino; and, P4,000 of accounts payable
are to be recorded in Al’s books. Their individual trial
balances, before adjustment, show the following:
Al
Bino
Assets P75,000
P113,000

47
Chapter 6 Variable vs.
Absorption Costing
Liabilities 5,000
34,500
Capital P70,000 P
78,500
Al’s capital, after adjustment, would be:
a. P65,000.
b. P66,000.
c. P68,500.
d. P70,000.
(PhilCPA)
125
. Lou, Mae, Nar, and Ovi, partners in a law firm, share
earnings in a 5:3:1:1 ratio, respectively. On July 1, their
relevant accounts follow:
Advances Loans
Capitals
(Dr) (Cr)
(Cr)
Lou P -0- P20,000
P160,000
Mae -0- 40,000
120,000
Nar 18,000 -0-
100,000
Ovi 10,000 -0-
60,000
On this day, cash of P72,000 was declared as available
for partners as profit distribution. Who, among the
partners, will benefit from the P72,000 cash
distribution?
a. Lou and Mae.
b. Mae and Nar.
c. Nar and Ovi.
d. All equally.
(PhilCPA)
126
. Ping and Pong are partners. During 2006, their capital
accounts on the partnership books appear as follows:
Ping
Pong
Opening balances P40,000
P20,000
Withdrawals 5,000
8,000

48
Multiple Choice Questions Variable vs. Absorption
Costing
Contributions 25,000
28,000
P60,000
P40,000

How much is Ping’s share in a net profit of P105,000 if


this is divided in the closing capital ratio?
a. P42,000.
b. P49,500.
c. P63,000.
d. P75,000.
(PhilCPA)
127
. On October 31, 2006, the balance sheet of the partners
Eva and Eda, who share profits and losses 3:2,
respectively, shows the following:

Sundry Assets
P600,000.
Eva, capital
360,000.
Eda, capital
240,000.

They agreed to take Ella as a new partner, with Ella


purchasing 1/8 of both partners’ interests for P100,000
cash. What amount would be recorded in the
partnership’s books as Ella’s capital, if the bonus
method is used?
a. P50,000.
b. P75,000.
c. P100,000.
d. P120,000.
(PhilCPA)
128
. When Nana retired from the partnership of Nana, Nina,
and Nona, the final settlement of Nana’s interest
exceeded her capital balance. Under the bonus
method, the excess is:
a. Recorded as goodwill.
b. Recorded as an expense.
c. Of no effect to the capital accounts of Nina and
Nona.

49
Chapter 6 Variable vs.
Absorption Costing
d. Deducted from the capital account balances of Nina
and Nona.
(PhilCPA)
129
. Ogie, Olie, and Orly are partners sharing profits and
losses 3:3:4, respectively. Orly gets permission to
withdraw from the partnership and they agree that
settlement shall be made by payments from personal
funds of the remaining partners. Their capital balances
are P30,000, P25,000, and P45,000, respectively, when
Orly withdraws. If Orly is paid P48,000 and the assets
revaluation method is used, the assets under-valuation
is:
a. P500.
b. P3,000.
c. P5,000.
d. P7,500.
(PhilCPA)
130
. Pedro and Pablo share partnership profits and losses in
a 7:3 ratio and their October 31, 2006 post closing trial
balance shows:

Cash
P 30,000
Accounts receivable
380,000
Inventory
260,000
Furniture
120,000
Accounts payable
(165,000)
Pedro, capital
(350,000)
Pablo, capital
(275,000)

Pekto offered to buy for P760,000 the partnership’s net


assets, except cash, after the assets are restated to
their current fair values as follows: accounts receivable,
P350,000; inventory, P250,000; and, furniture,
P135,000. How much will Pedro and Pablo receive as
final settlement of their partnership interest?

50
Multiple Choice Questions Variable vs. Absorption
Costing
a. P570,000.
b. P625,000.
c. P760,000.
d. P790,000.
(PhilCPA)
131
. Quinito, Quirino, and Quixote, partners, share profits in
the ratio of 4:2:1 and they have capital balances of
P11,200, P13,000, and P5,800, respectively. Prepare a
program showing how available cash will be distributed
as it becomes available, and indicate who, among the
partners, will be paid first with an available cash of
P1,400:
a. Quinito.
b. Quirino.
c. Quixote.
d. All – according to Profit & Loss ratio.
(PhilCPA)
132
. On May 31, 2007, Al, Ben, and Cip formed a
partnership by combining their businesses. Al gave
cash of P50,000. Ben gave a property with a carrying
amount of P30,000, an original cost of P40,000, and a
fair market value of P80,000. Ben’s property, however,
has a P35,000 mortgage for which the new partnership
accepted legal responsibility. Cip gave a delivery
equipment with a book value of P30,000, an acquisition
cost of P75,000, and an appraised value of P55,000. It
was agreed that profits and losses are to be shared
equally. The partner with the biggest capital account
balance as of May 31, 2007 is:
a. Al
b. Ben
c. Cip
d. All have equal capital balance
(PhilCPA)
133
. Delia and Ellen, on May 31, 2007, pooled their net
assets to form a partnership, with the new firm taking
over their business assets and assuming their liabilities.
The partners’ capitals are to be based on net assets
transferred after the following adjustments: allowance
for doubtful accounts of P1,000 and P1,500 are to be
set up on the books of Delia and Ellen, respectively;

51
Chapter 6 Variable vs.
Absorption Costing
Ellen’s inventory is to be increased by P3,000; and,
accounts payable of P4,000 is to be recorded on Delia’s
books. The individual trial balances on this date show:

Delia
Ellen
Assets P105,000
P113,000
Liabilities 35,000
34,500
Capital 70,000
78,500

What is Ellen’s adjusted capital balance?


a. P77,000.
b. P80,000.
c. P81,500.
d. P85,500
(PhilCPA)
134
. Paul, Quin, and Romy, partners in an accounting firm,
share profits in the respective ratio of 3:3:2. Saint is
admitted as a new partner and is allowed a 25% share
in profits, with the balance to be shared proportionately
by the original partners in their original ratio. What is
the new profit-sharing ratio of Paul, Quin, Romy, and
Saint, respectively?
a. 30%; 30%; 20%; 20%.
b. 40%; 40%; 10%; 10%.
c. 28.125%; 28.125%; 18.75%; 25%.
d. 37.5%; 37.5%; 25%; 25%.
(PhilCPA)
135
. Under their partnership agreement, Tina is to be
allowed a bonus of 20% of net income after bonus and
the remainder is to be divided as follows: 35% each to
Tina and Unna, and 30% to Vina. If the partnership’s
net income is P318,000, Tina’s share would be:
a. P79,500.
b. P92,750.
c. P132,500.
d. P145,750.
(PhilCPA)

52
Multiple Choice Questions Variable vs. Absorption
Costing
136
. The partnership agreement of Waldo, Xenon, and Yanni
provides for the following profit-sharing arrangement:
bonus of 20% of net income before bonus to Waldo;
interest at 15% on average capital balances; and any
remainder equally. During 2007, the partners
maintained average capital balances of P300,000,
P600,000, and P900,000, respectively. What is Waldo’s
share if the net income is P270,000?
a. P81,000.
b. P99,000.
c. P107,000.
d. P117,000.
(PhilCPA)

137
. The capital credit balances of the partners Edu and Fel
are P80,000 and P40,000, respectively, as of May 1,
2007. They share profits in the ratio of 3:2. They have
a desperate need for cash and they agree to admit Gus
as a new partner with a 1/3 interest in both capital and
profits upon the latter’s capital infusion of P30,000.
After the admission of Gus, assuming no goodwill is
recognized, the respective capital credit balances of
Edu, Fel,and Gus are:
a. P50,000, P50,000, and P50,000.
b. P66,667, P33,333, and P50,000.
c. P68,000, P32,000, and P50,000.
d. P80,000, P40,000, and P30,000.
(PhilCPA)
138
. Hugo, Ivan and Juni are partners sharing profits and
losses in the respective ratio of 3:3:4. Juni is given
permission to retire effective May 31, 2007, and it was
agreed that settlement is to be made by the remaining
partners making payment from their personal funds.
The capital balances on this date are P30,000,
P25,000, and P45,000 for Hugo, Ivan,and Juni,
respectively. If Juni received P45,000, how much did
Hugo pay Juni?
a. P13,500.
b. P18,000.
c. P22,500.
d. P45,000.

53
Chapter 6 Variable vs.
Absorption Costing
(PhilCPA)
139
. Karen, Karmi, and Kathy are partners sharing profits in
the respective ratio of 2:3:5. On May 31, 2007, Kathy
opted to retire. The capital account balances, at this
time are P95,000, P140,000, and P135,000,
respectively. Assuming that Kathy is paid P132,000,
Karen would be credited:
a. P600.
b. P857.
c. P1,200.
d. P1,800.
(PhilCPA)
140
. Tom, Umi, and Vic decided to dissolve their partnership
on May 31, 2007. On this date, their capital balances
and profit-sharing percents were as follows:

Tom P50,000
40%
Umi 60,000
30%
Vic 20,000
30%
The net income from January 1 to May 31, 2007 was
P44,000. Also on May 31, 2007, the partnership’s cash
and liabilities, respectively, were P40,000 and P90,000.
What was the book value of the partnership’s noncash
assets on May 31, 2007?
a. P180,000.
b. P190,000.
c. P220,000.
d. P224,000.
(PhilCPA)
141
. The condensed balance sheet and profit and loss
sharing ratio of the partnership of Wanda, Wendy, and
Wilma are presented below:
Cash P 22,500 Liabilities P
52,500
Due from Wanda 7,500 Due to Wilma
10,000
Other assets 205,000 Wand, capital (4)
75,000

54
Multiple Choice Questions Variable vs. Absorption
Costing
Irma, capital (3)
50,000
________ Irene, capital (3)
47,500
Total assets P 235,000 Total equities P
235,000
The partners agreed to liquidate and they sold all the
other assets for P150,000. How much of the available
cash should go to Wanda?
a. P42,500.
b. P45,500.
c. P53,000.
d. P75,000.
(PhilCPA)
142
. Dan, Ely, and Fil decided to dissolve their partnership on
May 31, 2007. On this date, their capital balances and
profit-sharing per cents were as follows:
Dan P50,000
40%
Ely 60,000
30%
Fil 20,000
30%
The net income from January 1 to May 31, 2007 was
P44,000. Also on May 31, 2007, the partnership’s cash
and liabilities, respectively, were P40,000 and P90,000.
For Dan to receive P55,200 in full settlement of his interest
in the partnership, how much must be realized from the
sale of the partnership’s non-cash assets?
a. P177,000.
b. P187,000.
c. P190,000.
d. P193,000.
(PhilCPA)
143
. Selected accounts of the KKK Partnership, just before
liquidation are shown below:

Kay, loan P
30,000 Dr.
Kim, loan
60,000 Cr.
Kay, capital
90,000 Dr.

55
Chapter 6 Variable vs.
Absorption Costing
Kim, capital
180,000 Cr.
Kuh, capital
150,000 Cr.

The partners share profits and losses at 20%, 40%, and


40%, respectively. In the liquidating program for cash
distribution to partners, the beginning capital balances
per unit of income share are
a. Kay: P –0- ; Kim: P120,000; Kuh: P75,000.
b. Kay: P(60,000); Kim: P120,000; Kuh: P75,000.
c. Kay: P(P120,000); Kim: P240,000; Kuh: P150,000.
d. Kay: P(240,000); Kim: P240,000; Kuh: P150,000.
(PhilCPA)
144
. Armida and Cheryl join in a partnership and agree to
divide initial capital equally, even though Armida
contributed P100,000 and Cheryl contributed P84,000
in identifiable assets. Under the bonus approach to
adjust capital accounts, Cheryl’s unidentifiable assets
should be debited for:
a. P-0-
b. P8,000.
c. P16,000.
d. P48,000.
(PhilCPA)
145
. On October 1, Eugene and Garri pooled their assets
and form a partnership, with the firm to take over their
business assets and assume their liabilities. The
partners’ capitals are to be based on net assets
transferred after the following adjustments: Garri’s
inventory is to be increased by P3,000; an allowance for
bad debts of P1,000 and P1,500 are to be set up in the
books of Eugene and Garri, respectively; and, P4,000
of accounts payable are to be recognized in Eugene’s
books. The individual trial balances on October 1 show
the following:
Eugene
Garri
Assets P113,000
P75,000
Liabilities 34,500
5,000

56
Multiple Choice Questions Variable vs. Absorption
Costing
Capital 78,500
70,000

What is the capital balance of Eugene and Garri


assuming they agree to share their capital equally?
a. P65,000.
b. P72,500.
c. P74,250.
d. P80,000.
(PhilCPA)
146
. The partnership agreement of Marie and Tonette allows
the former to received a 20% bonus on profits before
bonus and any residual profit/loss shall be divided 2:3,
respectively. Which partner has an advantage when the
partnership earns a profit or when it incurs a loss?
Profit Loss
a. Marie Marie
b. Tonette Tonette
c. Marie Tonette
d. Tonette Marie
(PhilCPA)
147
. Partners Mac, Ken, and Lee have the following profit-
sharing agreement: bonus of 25% of net income after
bonus to Mac; interest of 15% on average capital
balances; and, any remainder equally. Partners’
average capital balances are P300,000, P600,000, and
P900,000, respectively, for Mac, Ken, and Lee. If the
partnership’s net income is P270,000, Ken and Lee
would have respective shares of:
a. P72,000 and P117,000.
b. P81,000 and P72,000.
c. P90,000 and P135,000.
d. P92,000 and P138,000.
(PhilCPA)
148
. Tito and Vic, partners sharing profits and losses equally,
have capital balances of P90,000 each. Joey is
admitted as a new partner, making cash investment of
P120,000, to a one-third interest in both capital and
earnings. If Joey is credited in full for the amount of his
investment, the new capital of the partnership would be:
a. P240,000.

57
Chapter 6 Variable vs.
Absorption Costing
b. P300,000.
c. P360,000.
d. P420,000.
(PhilCPA)
149
. LEE, MON, and NED are partners who share profits
and losses equally and with capital balances of
P180,000 each. Lee retires from the partnership and
receives cash of P150,000. Assuming that the net
assets are fairly valued, the entry to record LEE’s
retirement is:
a. Lee, capital 180,000
Goodwill
30,000
Cash
150,000
b. Lee, capital 180,000
Other assets
30,000
Cash
150,000
c. Lee, capital 180,000
Mon, capital
15,000
Ned, capital
15,000
Cash
150,000
d. Lee, capital 180,000
Mon, capital 30,000
Ned, capital 30,000
Goodwill
90,000
Cash
150,000
(PhilCPA)
150
. Ana, Mae, and Rae share partnership profits and losses
in the ratio of 2:3:5, respectively. On October 31, 2007,
Rae was permitted to withdraw from the partnership at
which time their capital balances were:

Ana, capital
P25,000

58
Multiple Choice Questions Variable vs. Absorption
Costing
Mae, capital
40,000
Rae, capital
35,000

If Rae is paid P39,000 in full payment of her interest,


the capital of Ana immediately after Rae’s withdrawal
would be:
a. P22,600.
b. P23,000.
c. P23,400.
d. P26,600
(PhilCPA)

59
Chapter 6 Variable vs.
Absorption Costing
151
. Mar, Joe and Rey, who divide profits and losses 50%,
30%, and 20%, respectively, have the following October
31, 2007 account balances:

Mar, drawing (Dr.)


P12,000
Rey, drawing (Cr.)
4,800
Account receivable – Mar
7,200
Loans payable – Joe
14,400
Mar, capital
59,400
Joe, capital
44,400
Rey, capital
39,000

On this date, the partnership’s assets are P211,200


(including cash of P64,200). The partnership is
liquidated and Rey receives P33,000 in final settlement.
How much is the total loss on realization?
a. P10,800.
b. P31,200.
c. P54,000.
d. P64,200.
(PhilCPA)

END OF QUESTIONS

60
Multiple Choice Questions Variable vs. Absorption
Costing

61
1

SUGGESTED ANSWERS
. Letter “B” is the correct answer
Variable costing is the more accurate term. Variable (direct) costing considers
only variable manufacturing costs to be product costs, i.e., inventoriable.
However, these costs include variable manufacturing overhead, an indirect cost.
Answer (A) is incorrect because out-of-pocket costs refer to those requiring
immediate expenditure. Answer (C) is incorrect because relevant costs are
those that vary with alternative decisions. Answer (D) is incorrect because
prime costing includes only direct labor and direct materials costs (i.e., no
variable factory overhead).
2
. Letter “A” is he correct answer
Variable costing considers only variable manufacturing costs to be product
costs, i.e., inventoriable. Fixed manufacturing costs are treated as period costs.
Thus, one need only be able to determine the variable and fixed manufacturing
costs to institute a variable costing system.

Answer (B) is incorrect because even fixed costs are controllable in the long
run. Answer (C) is incorrect because standard costing is not necessary to
institute variable costing. Actual costs may be used. Answer (D) is incorrect
because selling prices as well as variable and fixed costs must be known to
calculate the contribution margin and breakeven point.

3
. Letter “B” is the correct answer
Product cost s under variable costing includes direct materials, direct labor, and
variable factory overhead. Each is a variable production cost.

Answer (A) is incorrect because variable factory overhead must also be


included. Answer (C) is incorrect because only variable production costs, not
variable selling and administrative costs, are product costs in variable costing.
Answer (D) is incorrect because absorption costing, not variable costing
includes all variable and fixed production costs.
4
. Letter “B” is the correct answer.
Variable costing inventory includes only variable manufacturing costs: direct
materials, direct labor, and variable factory overhead. Fixed factory overhead is
treated as a period cost.

Answer (A) is incorrect because variable factory overhead should be


included. Answer (C) is incorrect because variable –costing inventory costs
include direct labor and variable factory overhead, which are conversion cost.
Answer (D) is incorrect because conversion cost includes fixed factory overhead,
which is not a component of inventory under variable costing.
5
. Letter “C” is the correct answer.
Variable costing considers only variable manufacturing costs to be product
costs. Variable indirect cost s included in variable factory overhead is therefore
inventoriable. Fixed costs are considered period costs and are expense as
incurred.

Answer (A) is incorrect because variable manufacturing costs, whether


direct (direct materials and direct labor) or indirect (variable factory overhead),
are accounted for as a product costs, not period costs. Answer (B) is incorrect
because nonvariable indirect costs are treated as a period costs in variable
costing. Answer (D) is incorrect because, in variable costing, nonvariable direct
costs are treated as period costs, not product costs.
6
. Letter “D” is the correct answer.
Product costs are incurred to produce units of output, and they are expense
when the product is sold. Such costs include direct materials, direct labor, and
factory (not general and administrative) overhead. Period costs are charged to
expense as incurred because they are not identifiable with the product. Variable
costing considers only variable manufacturing costs to be product costs. Fixed
manufacturing costs and fixed and variable selling costs are considered period
costs and are expense as incurred.

Answer (A), (B), and (C) are incorrect because the fixed overhead and
selling costs are not identifiable with a product .
7
. Letter “A” is the correct answer.
Variable costing considers only variable manufacturing costs to be product
costs. Fixed manufacturing costs are period costs. Units -of -production
depreciation is included in variable factory overhead. Thus, a decrease in the
remaining useful life of machinery will increase the unit product cost.

Answer (B) is incorrect because variable cost per unit remains constant.
Answer (C) is incorrect because SYD depreciation affects fixed, not variable,
factory overhead. Answer (D) is incorrect because commissions are a selling
expense, i.e., a period cost, not a product cost.

8
. Letter “A” is the correct answer.
Activity-based costing, job-order costing, process costing, and standard costing
can all be used for both internal and external purposes. Variable costing is not
acceptable under GAAP for external-reporting purposes because it treats fixed
manufacturing costs as period costs.

Answer (B) is incorrect because job costing is acceptable for external


reporting. Answer (C) is incorrect because variable costing is acceptable for
internal purposes only. Answer (D) is incorrect because process costing is
acceptable for external reporting.
9
. Letter “D” is the correct answer.
Fixed costs are a basic expense of being in business; they are incurred to
continue operating the business regardless of production levels. Accordingly,
they are not controllable in the short run and should not be deferred.

Answer (A) is incorrect because period costs are controllable at higher


levels of management in the long run. Answer (B) is incorrect because period
costs are usually material. Answer (C) is incorrect because, although the
allocation of period costs may be arbitrary, the more basic rationale behind
variable costing is the lack of controllability in the short run.
10

. Letter “D” is the correct answer.


Variable costing treats fixed manufacturing costs as period costs, whereas
absorption costing accumulates them as product costs. If product costs are
viewed as all manufacturing costs incurred to produce output, fixed factory
overhead should be inventoried because it is necessary for production. The
counter argument in favor of variable costing is that fixed factory overhead is
more closely related to capacity to produce than to the production of individual
units. Internal reporting for cost behavior analysis is more useful if it
concentrates on the latter.
Answer (A) is incorrect because variable costing arguably understates
inventory. Answer (B) is incorrect because variable factory overhead is a product
cost under any cost system. Answer (C) is incorrect because the difficulty of
allocating fixed factory overhead is an argument against absorption costing.
11
. Letter “D” is the correct answer.
Gross profit (margin) is selling price minus CGS. The computation of CGS takes
into account fixed manufacturing overhead in inventory. Absorption costing
calculates gross profit. Variable costing treats fixed manufacturing overhead as
an expense in the period of incurrence. In variable costing, the contribution
margin (sales-variable costs) is calculated, not a gross profit (margin). Both
methods, however, compute operating income on their income statements.

Answer (A),(B), and (C) are incorrect because the variable-costing income
statement does not show gross profit, but it does include operating income.

12
. Letter “D” is the correct answer.
In a variable-costing income statement, the contribution margin equals sales
minus all variable costs, which include the variable selling and administrative
expenses as well as variable manufacturing costs (direct materials, direct labor,
and variable factory overhead). Operating income equals the contribution
margin minus all fixed costs.
Answer (A) and (C) are incorrect because variable selling and
administrative expenses are included in the determination of the contribution
margin. Answer (B) is incorrect because fixed selling and administrative
expenses are subtracted from the contribution margin to arrive at operating
income.

13
. Letter C is the correct answer.
Contribution margin is the difference between revenues and variable costs. No
distinction is made between variable product costs and variable selling costs;
both are deducted from revenue to arrive at CM.
Answer (A) is incorrect because CM is the excess of total variable costs,
not over fixed costs. Answer (B) is incorrect because projected revenues over
the breakeven point are the projected net income. Answer (D) is incorrect
because CM is the excess of total revenue over total variable costs, not variable
costs over fixed costs.
14
. Letter “B” is the correct answer.
Under the variable costing method, the contribution margin equals sales minus
variable expenses. Fixed selling and administrative costs and fixed factory
overhead are deducted from the contribution margin to arrive at operating
income. Thus, fixed costs are included only in the computation of operating
income.
Answer (A) is incorrect because fixed factory overhead is deducted from
the contribution margin to determine the operating income. Answer (C) is
incorrect because only variable expenses are used in the computation of the
gross margin. Answer (D) is incorrect because variable factory overhead is
included in the computation of contribution margin and fixed factory overhead
is not.

15
. Letter “B” is the correct answer.
In a variable costing system, only the variable costs are recorded as product
costs. All fixed costs are expensed in the period incurred. Because changes in
the relationship between production and sales do not cause changes in the
amount of fixed manufacturing cost that is expensed, profits more directly
follow the trends in sales.
Answer (A) is incorrect because, in variable costing, fixed costs are
charged as period costs and cannot cause a change in unit cost as production
increases. Answer (C) is incorrect because idle facility variation is calculated
under an absorption costing system. Answer (D) is incorrect because neither
variable nor absorption costing includes administrative costs in inventory.
16
. Letter “C” is the correct answer.
Absorption (full) costing considers all manufacturing costs to be inventoriable as
product costs. These costs include variable and fixed manufacturing costs,
whether direct or indirect. The alternative to absorption costing is variable
(direct) costing.
Answers (A) and (B) are incorrect because variable (direct) costing does
not inventory fixed factory overhead. Answer (D) is incorrect because
conversion costs include direct labor and factory overhead but not direct
materials.
17
. Letter “D” is the correct answer.
Using absorption costing, fixed manufacturing overhead is included in
inventoriable (product) costs. Fixed manufacturing overhead costs are indirect
costs because they cannot feasibly be directly traced to specific units produced.
Answers (A),(B), and (C) are incorrect because fixed manufacturing
overhead costs are neither direct nor period costs under absorption costing.
18
. Letter “C” is the correct answer.
In an absorption costing system, fixed overhead costs are included in the
inventory. When sales exceed production, more overhead is expensed under
absorption costing due to fixed overhead carried over the prior inventory. If
sales increase over production, more than one period’s factory overhead is
recognized as expense. Accordingly, if the increase in factory overhead
expensed is greater than the contribution margin of the increased units sold,
there may be less profit with an increased level of sales.
Answers (A) and (B) are incorrect because profit is a function of both
sales and production, so it will not always move in the same direction as sales.
Answer (D) is incorrect because decreased output will increase the unit cost of
items sold. Fixed factory overhead per unit will increase.

19
. Letter “B” is the correct answer.
Under variable costing, inventories are charged only with the variable costs of
production. Fixed manufacturing costs are expensed as period costs. Absorption
costing charges to inventory all costs of production. If finished goods inventory
increases, absorption costing results in higher income because it capitalizes
some fixed costs that would have been expensed under variable costing. When
inventory declines, variable costing results in higher income because some fixed
costs capitalized under the absorption method in prior periods are expensed in
the current period.
Answer (A) is incorrect because fixed factory overhead is treated
differently under the two methods. Answer (C) is incorrect because variable
costs are the same under either method. Answer (D) is incorrect because gross
margins will be different. Fixed factory overhead is expensed under variable
costing and capitalized under the absorption method.

20
. Letter “D” is the correct answer.
Under absorption costing, all manufacturing costs are treated as product costs.
Under variable costing, only variable costs of manufacturing are inventoried as
product costs. Fixed manufacturings costs are expensed as period costs.
Packaging and shipping costs are not product costs under either method
because they are incurred after the goods have been manufactured. Instead,
they are included in selling and administrative expenses for the period.
Answers (A) and (C) are incorrect because manufacturing supplies and
direct labor are variable costs inventoried under both methods. Answer (B) is
incorrect because factory insurance is a fixed manufacturing cost inventoried
under absorption costing but written off as period cost under variable costing.

21
. Letter “C” is the correct answer.
Gross margin equals sales minus CGS (BI + CGM – EI). An
absorption costing system applies fixed as well as variable factory overhead to
products. Because Blue’s production has always been less than planned activity,
fixed overhead was underapplied each year. Hence, Blue must have debited
underapplied fixed factory overhead (an unfavorable production volume
variance) each year to CGS, WIP, and FG. Because production always equaled
sales, however, no inventories existed at any year-end, and annual
underapplication should have been debited entirely to CGS. Consequently, the
gross margins for years 1 and 3 must be the same because the gross revenue
and CGS were identical for the two periods.
Answer (A) is incorrect because the gross margins for years 1 and 3 are
equal. Answers (B) and (D) are incorrect because the greater sales volume in
year 2 should have produced a greater gross margin than in year 1 or 3.

22
. Letter “C” is the correct answer.
The gross margin percentage equals gross profit (sales – CGS) divided by sales.
Sales are given as P40, 000, and expenses included in cost of goods sold are
listed below. The gross margin is P18, 400, which is 46% of P40, 000.

Sales P40,000
Cost of goods sold
Direct materials P9,050
Direct labor 6,050
Rent (9/10 x P3000) 2,700
Depreciation 2,000
Supervision (2/3 x P1,500) 1,000
Insurance (2/3 x P1,200) 800 (21,600) P18,400

Office expenses are usually general and administrative expenses, which are
period rather than product costs.
Answer (A) is incorrect because 41% results from including sales salaries
and advertising expenses in the calculation. Answer (B) is incorrect because
44% results from including 100% of the rent and supervision expenses. Answer
(D) is incorrect because 51% omits depreciation on factory equipment from the
calculation.

23
. Letter “B” is the correct answer.
The total allocations to the partners as a result of the interest salary, and bonus
and the over allocations amount to P83,000 and P23,000, computed as follows:
Reynold Serena Total
Interest (10%) P20,000 P30,000 P50,000
Bonus (5% x P60,000) 3,000 3,000
Salary 10,000 20,000 30,000
Total Allocations P30,000 P53000 P83,000
Net Income 60,000
Over-allocations P23,000
Take note that the interest is based on the capital balance on January 1, not on
the weighted average capital balance. Bonus of 5% is given because the net
income after salary exceeds P20,000 (P60,000 income – P30,000 salary
allocation).
24
. Letter “A” is the correct answer.
If instead of net income, the partnership incurred a net loss of P20,000, each
partner’s capital will be reduced by P10,000 each, computed as follows:
Reynold Serena Total
Interest (10%) P20,000 P30,000 P50,000
Salary 10,000 20,000 30,000
Total Allocations P30,000 P50000 P80,000
Less over-allocations (4:6) 40,000 60,000 100,000
Share in the loss (P10,000) (P10,000) (P20,000)
Take note that bonus is not allocated because there is a loss. The over-
allocation is equal to the P80,000 total amount of interest and salary plus the
actual net loss of P20,000.
25
. Letter “C” is the correct answer.
The capital accounts of Able, Bable and Cable shall be increased by their
respective share in the net appreciation of the value of the assets of the
partnership amounting to P180,000 which is computed as follows:

Fair value of tangible assets:


Land P200,000
Marketable securities 250,000 P450,000
Less book value (P100,000 + P200,000) 300,000
Appreciation in value P150,000
Add goodwill to be recorded 30,000
Total increase in capital P180,000
Increase in the capital of each partner:
Able – P180,000 x 2/10 = P36,000
Bable – P180,000 x 3/10 = P54,000
Cable – P180,000 x 5/10 = P90,000
26
. Letter “C” is the correct answer.
In order to have a one-fifth interest, Daboy must invest an amount equal to
P301,250, computed as follows:
Capital of the old partners before adjustments P1,025,000
Add total increase in capital 180,000
Adjusted capital of the old partnership P1,205,000
Divided by old partners capital ratio (4/5) or 80%
Total required capitalization P1,506,250
Multiplied by capital interest of Daboy (1/5) or 20%
Required contribution of Daboy P 301,250
27
. Letter “D” is the correct answer.
The new profit sharing ratio of the partners assuming the old partners continue
to share in their old ratio shall be 80% for the old partners Able, Bable, and
Cable and 20% for Daboy. The 80% or 4/5 of the profit shall be allocated among
the old partners in the ratio of 2:3:5 or 16% (20 x 80%) for Able, 24% (30% x
80%) for Bable, 40% (50% x 80%) for Cable and 20% for Daboy. To determine
the correct answer from the given choices, each must be converted into
percentage. Choice letter D is equivalent to this answer.
Letter A is incorrect because the ratio is equivalent to 20%, 30%, 30% and 20%.
Letter B is incorrect because the ratio is equivalent to 17%, 22%, 48% and 23%.
Letter C is incorrect because the ratio is equivalent to 18%, 27%, 27% and 27%.
28
. Letter “B” is the correct answer.
In order to have an 80% interest, Daboy must invest an amount equal to
P4,820,000, computed as follows:
Capital of the old partners before adjustments P1,025,000
Add total increase in capital 180,000
Adjusted capital of the old partnership P1,205,000
Divided by old partners capital ratio (100%-80%) 20%
Total required capitalization P6,025,000
Multiplied by capital interest of Daboy (1/5) or 80%
Required contribution of Daboy P4,820,000
29
. Letter “C” is the correct answer.
The book value of the partnership equity (net assets) on June 30, 2005 is equal
to P145,000. This amount is computed as follows:
Rita, capital P70,000
Less loan to Rita 30,000 P 40,000
Sally, capital P50,000
Add loan from Sally 25,000 75,000
Tracy, capital 30,000
Total partnership equity (net assets) P 145,000
30
. Letter “D” is the correct answer.
The cash available for distribution to partners on July 31, 2005 is equal to
P10,000, computed as follows:
Cash as of June 30, 2005 P20,000
Add: Collection on receivables P 15,000
Sale of inventory 20,000 35,000
Total P55,000
Less accounts payable P35,000
Liquidation expenses 10,000 45,000
Cash available for distribution to partners P10,000
31
. Letter “A” is the correct answer.
A safe payment plan must be prepared in order to determine the proper distribution of
any cash available. Any amount set aside for liquidation expenses shall be considered
as additional loss on realization of non-cash assets.
Rita Sally Tracy
Capital before realization of assets P70,000 P50,000 P30,000
Less share on realization loss:
Accounts receivable:
(P30,000 – 15,000) x 20% (3,000)
(P30,000 – 15,000) x 30% (4,500)
(P30,000 – 15,000) x 50% (7,500)
Inventory:
(P70,000 – 20,000) x 20% (10,000)
(P70,000 – 20,000) x 30% (15,000)
(P70,000 – 20,000) x 50% (25,000)
Plant assets:
(P60,000 x 20%) (12,000)
(P60,000 x 30%) (18,000)
(P60,000 x 50%) (30,000)
Liquidation expenses:
P10,000 x 20% (2,000)
P10,000 x 30% (3,000)
P10,000 x 50% ( 5,000)
Balance P43,000 P 43,000 P 9,500 (P37,500)
Deficiency of Tracy (2:3) (15,000) ( 22,500) 37,500
Balance P 28,000 (P13,000) - Add
(deduct) loan from (to) (30,000) 25,000
Balance (P 2,000) P12,000
Offset of Rita’s deficiency 2,000 ( 2,000)
Amount to be received - P10,000

32
. Letter “C” is the correct answer.
A cash priority program is necessary in order to determine the answer to this
question. The following is the cash priority program for the partnership of Ana,
Beta, and Cynthia:
Ana (2)Beta (1)Cynthia (1)Capital before liquidationP100,000P50,000P50,000Divided by P & L
ratio2/41/41/4Loss absorption capacityP200,000P200,000P200,000
Because the loss absorption capacity of the partners is the same, any cash
available to the partners will be distributed according to the profit and loss
sharing ratio as it becomes available.
33
. Letter “A” is the correct answer.
A cash priority program is needed in order to find out how any cash available to
the partners shall be distributed after outside creditors are paid. The following
schedule meets that requirement:
Bre –25%Gil –25%Da – 50%Capital before liquidationP70,000P80,000P150,000Loan from (to)
partners (40,000)50,000Net interest bef. liquidationP30,000P130,000Divided by P & L ratio
25% 25% 50%Loss absorption capacityP120,000P520,000P300,000First
priority220,000Second priority180,000180,000P120,000P120,000P120,000
The first priority in the cash distribution belongs to Gil. Before any partner
receive any amount, Gil must first get P55,000 (25% x P220,000). Any amount
available in excess of P55,000 but not exceeding P190,000 shall be divided
between Gil and Da in the ratio of 1:2, respectively.
34
. Letter “D” is the correct answer.
The amount of cash to be received by Beta in his first distribution is equal to
P147,000, computed as follows:
AlphaBeta CharlieCapital before liquidationP135,000P216,000P49,000Assumed loss on assets:
P200,000 x 40%
P200,000 x 30%
(80,000)

(P60,000)

(P60,000)Share on contingency fee:


P10,000 x 40%
P10,000 x 30%
(4,000)

(3,000)
(3,000)BalanceP51,000P153,000(P14,000)Less share in C’s deficiency:
P14,000 x 4/7
P14,000 x 3/7
(8,000)

(6,000)
8,000
6,000Safe payment to partnersP43,000P147,000-
35
. Letter “A” is the correct answer.
If the partners agree to distribute available cash Fox will receive P23,000 out of
the cash distribution. The allocation of cash is determined as follows:
Delta (4)Echo (4)Fox (2)Capital before liquidationP79,000P140,000P140,000Less loss
on inventory(8,000)(8,000)(4,000)BalanceP71,000P132,000P136,000Less loss on PPE
(4:4:2)(92,000)(92,000)(46,000)Balance(P21,000)P40,000P30,000Share in Delta’s
Deficit 21,000(14,000)7,000Distribution of cash-P26,000P23,000
36
. Letter “C” is the correct answer.
If a partnership has income of P44,000 and Partner Alpha is to be allocated a
bonus of 10% of income after bonus, then Partner Alpha shall receive a bonus
equal to P4,000, computed as follows:
Let B = Bonus, then B = 10% (P44,000 – B)
B = P4,400 – 10%B
110% B = P4,400
B = P4,400/110%
B = P4,000
37
. Letter “B” is the correct answer.
The weighted average capital balance of the partner if withdrawals in excess of
P10,000 are charged to the partner’s capital account amount to P51,667,
computed as follows:
Initial contribution, March 1 P50,000 x 12/12 = P41,667
Additional contribution, June 1 P20,000 x 7/12 = 11,667
Withdrawal exceeding P10,000 P5,000 x 4/12 = (1,667)
Weighted average capital P51,667
38
. Letter “C” is the correct answer.
The share of Arthur in partnership income is equal to P70,000, computed as
follows:
Ending capital balance P 72,000
Add back withdrawal (P5,000 x 12) 60,000
Balance P132,000
Less: Additional investment P27,000
Beginning capital balance 35,000 62,000
Share in annual partnership income P 70,000
39
. Letter “B” is the correct answer.
If the partnership had net income of P102,500, the amount to be allocated to
Jack is equal to P44,250, computed as follows:
JackJillTotalSalariesP30,000P45,000P75,000Bonus2,500-
2,500Interest2,0003,5005,500Balance9,7509,75019,500TotalP44,250P58,250P102,500
40
. Letter “B” is the correct answer.
The first P100,000 of available cash in liquidation shall be used to pay outside
liabilities amounting to P70,000 and the P30,000 balance paid to D based on the
following cash priority program:
D E F
Capital before liquidation P70,000 P 30,000 P 50,000
Loans payable to D 20,000 _______ _______
Total interest P90,000 P 30,000 P 50,000
Divided by capital ratio 1/3 1/3 1/3__ Loss absorption
capacity P270,000 P 90,000 P150,000
D E F
1st priority to:
D - 1/3 x (P270,000-P150,000) =P40,000
2nd priority to:
D – 1/3 (P150,000 – P90,000) = 20,000
F - 1/3 (P150,000 – P90,000) = _______ 20,000
Total distribution P60,000 P20,000

Since D owns the first priority in cash distribution (P40,000) then he is the only
one entitled to receive the P30,000 cash available to the partners.
41
. Letter “D” is the correct answer.
The amount of assets available to partners after paying all the liabilities
(including loan payable to G) and P30,000 to I is P75,000 (P195,000 – P70,000 –
P20,000 – P30,000) and this amount is to be distributed P55,000, P15,000 and
P5,000 to G, H, and I, respectively. This distribution is based on the following
safe payment plan:

D E F
Capital before liquidation P70,000 P 30,000 P 50,000
Less realization loss* (15,000) (15,000) (15,000)
Balance P55,000 P 15,000 P 35,000
Less amount received - - (30,000) Cash distributed
P55,000 P 15,000 P 5,000

42
. Letter “A” is the correct answer.
The amount of cash to be received by Elton in addition to the machine before
any of the other partners received anything shall be equal to P15,000,
computed as follows:

Dalton Elton Falcon


Capital before liquidation P 40,000 P 90,000 P30,000
Divided by capital ratio 1/3 1/3 1/3_ Loss
absorption capacity P120,000 P270,000 P90,000

First priority payment to Elton:


1/3 x (P270,000 – P120,000) = P50,000

Less fair value of machine (35,000)


Amount of cash in first priority payment P15,000
43
. Letter “B” is the correct answer.
The capital balances of Abra, Cada, and Bravo after the admission of Debra shall
be P85,000, P65,000 and P50,000, respectively, computed as follows:
Abra Cada Bravo
Capital before goodwill P 80,000 P 60,000 P45,000
Share in goodwill* 5,000 5,000 5,000
Capital after admission P 85,000 P 65,000 P50,000

* Net assets of old partners per appraisal P200,000


Capital contributions (P80,000 + P60,000 + P45,000) 185,000
Goodwill P 15,000
Divided by (equally) 3
Share in goodwill P 5,000
44
. Letter “B” is the correct answer.
The partner most vulnerable (susceptible) to partnership losses on liquidation is
Ring, the partner who has the least loss absorption capacity based on the
following cash priority program:

Queen Ring Sting


Capital before liquidation P 45,000 P 30,000 P 15,000
Add (deduct) loan from (to) ( 5,000) 10,000
Total partner’s interest P 45,000 P 25,000 P 25,000
Divided by P/L ratio 30% 50% 20% _
Loss absorption capacity P150,000 P 50,000 P125,000
Vulnerability Rank 3rd 1st 2nd
45
. Letter “D” is the correct answer.
If P65,000 cash is available for distribution, it should be paid P50,000 to outside
liabilities, P12,000 to Queen and P3,000 to Sting. Ring receives nothing
because he is the least in the priority for cash distribution based on the
following cash priority program:

Queen Sting
First priority payment to Queen:
30% x (P150,000 – P125,000) = P 7,500

Second priority payment to Queen & Sting:


30% x (P125,000 – P50,000) = 22,500
20% x (P125,000 – P50,000) = P15,000
Total priority payment P30,000 P15,000

Any amount in excess of P45,000 (P30,000 + P15,000) shall be divided among


the partners based on their profit and loss sharing ratio. However, because the
cash available to the partners is only P15,000 (P65,000 total cash available –
P50,000 outside liabilities), P7,500 shall be given to Queen and the P7,500
balance (P15,000 – P7,500) will be shared by Queen and Sting based on their
profit/loss ratio of 30:20. Hence, Queen receives P12,000 [P7,500 + 60%
(P7,500)], and Sting receives P3,000 (40% x P7,500).
46
. Letter “A” is the correct answer.
Based on the cash priority program in No. 57, if only P7,500 is available for
distribution to partners after paying all non-partner liabilities, then it should be
distributed to Queen only, Ring and Sting receives nothing.
47
. Letter “C” is the correct answer.
The available cash should be distributed P25,000 to accounts payable, P1,000 to
Maris and P8,000 to Tela. The debit balance in the capital of Maris is to be offset
against her loan balance. This may also be computed as follows:
Maris Tela Total
Capital before liquidation P(8,000 P 8,000 P -0-
Loan balance 9,000 9,000 Amount
Receivable P 1,000 P 8,000 P9,000
48
. Letter “D” is the correct answer.
If available cash is distributed immediately, Bush should get P15,000 and Sadam,
P5,000 based on the following safe payment plan:
George Bush Sadam
Capital before liquidation P 40,000 P 45,000 P35,000
Less share in assumed loss on inventories
(50:25:25) (50,000) (25,000)
(25,000)
Balance P (10,000) P 20,000
P10,000
Share on deficiency (25:25) 10,000
(5,000) (5,000)
Cash received P ----0------ P 15,000 P 5,000

49
. Letter “C” is the correct answer.
The available cash should be distributed P70,000 to Friendly and P5,000 to Healthy,
respectively. Dearly receives nothing based on the following safe payment plan:
Dearly Friendly Healthy
Capital before liquidation P 75,000 P200,000 P155,000
Loss on noncash assets* ( 69,000) ( 138,000) ( 138,000)
Balance P 6,000 P 62,000 P 17,000
Less liquidation expenses (1,000) (2,000) (2,000)
Balance P 5,000 P 60,000 P 15,000
Add loan from (to) (25,000) 20,000 _______
Balance P(20,000) P 80,000 P 15,000
Less offset of deficiency 20,000 (10,000) (10,000)
Amount to be received P -0- P 70,000 P 5,000
50
. Letter “C” is the correct answer.
The amount of Unity’s partnership equity that appear to be recoverable is
P79,000, computed as follows:

Unity Vanity Witty


Capital before liquidation P 90,000 P (60,000) (P100,000)
Contribution by Witty ________ _________ 70,000_
Balance P 90,000 P (60,000) (P 30,000)
Contribution by Vanity ________ 39,000 _________
Balance P 90,000 P (21,000) (P 30,000)
Share in Vanity’s deficit ( 9,000) 21,000 ( 12,000)
Balance P 81,000 P -0- (P 42,000)
Less offset of deficit ________ ________ 40,000
Balance P 81,000 P -0- ( 2,000)
Less Witty’s deficit (2,000) ________ 2,000
Amount recoverable P 79,000 P -0- P -0-
51
. Letter “B” is the correct answer.
Upon the admission of Vince, goodwill amounting to P15,000 shall be recorded,
computed as follows:
Total agreed capital (given) P330,000
Total contributed capital (P92,000+95,000 + P68,000 +
P60,000) 315,000
Goodwill to be credited to old partners P 15,000
52
. Letter “C” is the correct answer.
After the admission of Vince, Marc’s capital balance would be, P79,100,
computed as follows:
Vince Mitz (5) Marc (3) Mert (2)
Capital Contributions of Partners
P80,000 P95,000
P80,000 P60,000
Capital bought (sold) 12,000
(12,000) -___
Total contributions P92,000 P95,000
P68,000 P60,000
Bonus to old partners (22,000)
11,000 6,600 4,400
Capital balance P70,000
P106,000 P74,600
P64,400
Goodwill (#63) to old partners (5:3:2)
7,500 4,500
3,000
Capital balance P113,500
P79,100
P67,400
53
. Letter “B” is the correct answer.
The amount of cash to be received by John in the cash distribution is equal to
P20,000, computed as follows:

Alex Jay John


Capital before liquidation P 95,000 P 80,000 P70,000
Less share in realization loss* 97,500 48,750 48,750
Balance P (2,500) P 31,250 P21,250
Offset of capital deficiency 2,500 1,250 1,250
Amount to be received P -0- P 30,000 P20,000
* P265,000 – P70,000 = P195,000 x 50% = P97,500
P195,000 x 25% = P48,750
54
. Letter “B” is the correct answer.
The capitals of Ana, Bena and Cena, after the admission of Dina shall be,
P210,00, P126,000, and P84,000, respectively, computed as follows:
Ana Bena Cena
Capital before admission P250,000 P150,000 P100,000
Add goodwill share (5:3:2) 30,000 18,000 12,000
Capital after goodwill P280,000 P168,000 P112,000
Less capital sold (25%) (70,000) (42,000) (28,000)
Capital after admission P210,000 P126,000 P 84,000
55
. Letter “D” is the correct answer.
If profits are to be divided based on weighted average capital, the share of
Annie and Bennie shall be equal to P136,543 and P103,457, respectively,
computed as follows:
Share of Annie: P240,000 x P108,334*/190,417* = P136,543
Share of Bennie: P240,000 – P136,543 = P103,457
*Weighted average capital:
Annie Bennie
P120,000 x 12/12 = P120,000
P80,000 x 12/12 = P80,000
P20,000 x 8/12 = (13,333)
P10,000 x 8/12 = (6,667)
P20,000 x 6/12 = 10,000
P10,000 x 5/12 = 4,167
P10,000 x 3/12 = (2,500)
P5,000 x 3/12 = (1,250)
Weighted average capital P108,334 P82,083

Total (P108,334 + P82,083) P190,417


56
. Letter “A” is the correct answer.
If 20% interest based on capital at the end of the year is to be allowed, and then
the balance of the profit is to be divided equally, the total share of Annie and
Bennie shall be equal to P121,500 and P118,500, respectively, computed as
follows::

Annie Bennie Total


Interest on ending capital* P 20,000 P 17,000 P 37,000
Balance divided equally** 101,500 101,500 203,000
Total P121,500 P118,500 P240,000
Ending capital balance:
Annie: P120,000 –P20,000 + P10,000 – P10,000 = P100,000
Bennie: P80,000 – P10,000 + P20,000 – P5,000 = P85,000
*Interest on ending capital balance:

Annie: 20% x P100,000 = P20,000


Bennie: 20% x P85,000 = P17,000

** P240,000 – P37,000 = P101,500

57
. Letter “A” is the correct answer.
Zerex should contribute assets amounting to P35,000, computed as follows:
Total Capital of partnership before admission P140,000
Divided by their total capital ratio (5/5 – 1/5) 4/5 Total agreed
capitalization P175,000
Multiplied by Zerex interest 1/5
Amount of assets to be contributed by Zerex P 35,000
58
. Letter “D” is the correct answer.
If instead of admission of a new partner, the partnership is liquidated by
installments, after payment of liabilities, the available cash shall be distributed
P24,000, P13,000 and P13,000 to Werex, Xerex and Yerex, respectively, computed
as follows:

Werex Xerex Yerex


Capital before liquidation P 60,000 P 40,000 P40,000
Less realization loss share* 36,000 27,000 27,000 Cash
distribution P 24,000 P 13,000 P13,000
* P130,000 – 40,000 = P90,000 x 40% = P36,000
P90,000 x 30% = P27,000
59
. Letter “A” is the correct answer.
The bonus of Mano is equal to 15% of net profit after adding back the amount of
salary, interest and bonus. The P32,000 net profit is computed after deducting the
salary, interest, and bonus. After adding back the salary and interest, the amount
computed is equal to the net profit before salary and interest but after bonus. Since
bonus is equal to 15% then the net profit before salary and interest is equal to 85%
(100% - 15%). The bonus is therefore equal to P18,000, computed as follows:
Net Profit P 32,000
Add salary (P5,000 x 12) 60,000
Net profit before salary but after interest P 92,000
Add interest (P175,000 + P25,000) x 5% 10,000
Net profit before salary and interest but after bonus P102,000
Divided by ratio net of bonus (100% - 15%) 85%
Net profit before salary, interest and bonus P120,000
Multiplied by bonus rate 15%
Bonus of Mano P 18,000

60
. Letter “C” is the correct answer.
The remaining cash amounting to P80,000 (P30,000 + P100,000 – P50,000),
shall be distributed P-0-, P31,000, and P49,000 to Salve, Gilda, and Nora,
respectively, according to the following safe payment plan:
Salve (5) Gilda (3) Nora (2)
Capital before liquidation P 80,000 P115,000 P105,000
Less realization loss* (5:3:2) (110,000) 66,000 44,000
Balance P(30,000)P 49,000 P 61,000
Offsetting of deficit 30,000 (18,000) (12,000)
Cash distribution P –0- P 31,000 P 49,000
* Realization loss = P320,000 – P100,000 = P220,000
Salve: P220,000 x 50% = P110,000
Gilda: P220,000 x 30% = P 66,000
Nora: P220,000 x 20% = P 44,000
61
. Letter “D” is the correct answer.
The P176,000 operating income of the partnership shall be divided as follows :
Herman Marco Alexis Total
Salary P 24,000 P 18,000 P12,000 P 54,000
Interest (12%)* 19,500 24,700 29,400 73,600
Balance (2:4:4)** 9,680 19,360 19,360 48,400
Total share P 53,180 P 62,060 P60,760 P176,000

Weighted average capital of Herman Marco Alexis


P150,000 x 12/12 P150,000
P30,000 x 6/12 15,000
P10,000 x 3/12 ( 2,500)
P200,000 x 12/12 P200,000
P20,000 x 5/12 8,333
P10,000 x 3/12 (2,500)
P250,000 x 12/12 P250,000
P30,000 x 2/12 (5,000)
Weighted average capital P162,500 P205,833 P245,000
62
. Letter “A” is the correct answer.
The amount of cash to be received by Elton in addition to the machine before
any of the other partners received anything shall be equal to P15,000,
computed as follows:

Dalton Elton Falcon


Capital before liquidation P 40,000 P 90,000 P30,000
Divided by capital ratio 1/3 1/3 1/3_ Loss
absorption capacity P120,000 P270,000 P90,000

First priority payment to Elton:


1/3 x (P270,000 – P120,000) = P50,000

Less fair value of machine (35,000)


Amount of cash in first priority payment P15,000
63
. Letter “A” is the correct answer.
The amount of cash to be received by Elton in addition to the machine before
any of the other partners received anything shall be equal to P15,000,
computed as follows:

Dalton Elton Falcon


Capital before liquidation P 40,000 P 90,000 P30,000
Divided by capital ratio 1/3 1/3 1/3_ Loss
absorption capacity P120,000 P270,000 P90,000

First priority payment to Elton:


1/3 x (P270,000 – P120,000) = P50,000

Less fair value of machine (35,000)


Amount of cash in first priority payment P15,000
64
. Letter “A” is the correct answer.
The amount of cash to be received by Elton in addition to the machine before
any of the other partners received anything shall be equal to P15,000,
computed as follows:

Dalton Elton Falcon


Capital before liquidation P 40,000 P 90,000 P30,000
Divided by capital ratio 1/3 1/3 1/3_ Loss
absorption capacity P120,000 P270,000 P90,000

First priority payment to Elton:


1/3 x (P270,000 – P120,000) = P50,000

Less fair value of machine (35,000)


Amount of cash in first priority payment P15,000
65
. Letter “A” is the correct answer.
The amount of cash to be received by Elton in addition to the machine before
any of the other partners received anything shall be equal to P15,000,
computed as follows:

Dalton Elton Falcon


Capital before liquidation P 40,000 P 90,000 P30,000
Divided by capital ratio 1/3 1/3 1/3_ Loss
absorption capacity P120,000 P270,000 P90,000

First priority payment to Elton:


1/3 x (P270,000 – P120,000) = P50,000

Less fair value of machine (35,000)


Amount of cash in first priority payment P15,000
66
. Letter “A” is the correct answer.
The amount of cash to be received by Elton in addition to the machine before
any of the other partners received anything shall be equal to P15,000,
computed as follows:

Dalton Elton Falcon


Capital before liquidation P 40,000 P 90,000 P30,000
Divided by capital ratio 1/3 1/3 1/3_ Loss
absorption capacity P120,000 P270,000 P90,000

First priority payment to Elton:


1/3 x (P270,000 – P120,000) = P50,000

Less fair value of machine (35,000)


Amount of cash in first priority payment P15,000
67
. Letter “A” is the correct answer.
The amount of cash to be received by Elton in addition to the machine before
any of the other partners received anything shall be equal to P15,000,
computed as follows:

Dalton Elton Falcon


Capital before liquidation P 40,000 P 90,000 P30,000
Divided by capital ratio 1/3 1/3 1/3_ Loss
absorption capacity P120,000 P270,000 P90,000

First priority payment to Elton:


1/3 x (P270,000 – P120,000) = P50,000

Less fair value of machine (35,000)


Amount of cash in first priority payment P15,000
68
. Letter “A” is the correct answer.
The amount of cash to be received by Elton in addition to the machine before
any of the other partners received anything shall be equal to P15,000,
computed as follows:

Dalton Elton Falcon


Capital before liquidation P 40,000 P 90,000 P30,000
Divided by capital ratio 1/3 1/3 1/3_ Loss
absorption capacity P120,000 P270,000 P90,000

First priority payment to Elton:


1/3 x (P270,000 – P120,000) = P50,000
Less fair value of machine (35,000)
Amount of cash in first priority payment P15,000
69
. Letter “A” is the correct answer.
The amount of cash to be received by Elton in addition to the machine before
any of the other partners received anything shall be equal to P15,000,
computed as follows:

Dalton Elton Falcon


Capital before liquidation P 40,000 P 90,000 P30,000
Divided by capital ratio 1/3 1/3 1/3_ Loss
absorption capacity P120,000 P270,000 P90,000

First priority payment to Elton:


1/3 x (P270,000 – P120,000) = P50,000

Less fair value of machine (35,000)


Amount of cash in first priority payment P15,000
70
. Letter “A” is the correct answer.
The amount of cash to be received by Elton in addition to the machine before
any of the other partners received anything shall be equal to P15,000,
computed as follows:

Dalton Elton Falcon


Capital before liquidation P 40,000 P 90,000 P30,000
Divided by capital ratio 1/3 1/3 1/3_ Loss
absorption capacity P120,000 P270,000 P90,000

First priority payment to Elton:


1/3 x (P270,000 – P120,000) = P50,000

Less fair value of machine (35,000)


Amount of cash in first priority payment P15,000

71
. Letter “A” is the correct answer.
The amount of cash to be received by Elton in addition to the machine before
any of the other partners received anything shall be equal to P15,000,
computed as follows:

Dalton Elton Falcon


Capital before liquidation P 40,000 P 90,000 P30,000
Divided by capital ratio 1/3 1/3 1/3_ Loss
absorption capacity P120,000 P270,000 P90,000

First priority payment to Elton:


1/3 x (P270,000 – P120,000) = P50,000

Less fair value of machine (35,000)


Amount of cash in first priority payment P15,000
72
. Letter “A” is the correct answer.
The amount of cash to be received by Elton in addition to the machine before
any of the other partners received anything shall be equal to P15,000,
computed as follows:

Dalton Elton Falcon


Capital before liquidation P 40,000 P 90,000 P30,000
Divided by capital ratio 1/3 1/3 1/3_ Loss
absorption capacity P120,000 P270,000 P90,000

First priority payment to Elton:


1/3 x (P270,000 – P120,000) = P50,000

Less fair value of machine (35,000)


Amount of cash in first priority payment P15,000

73
. Letter “A” is the correct answer.
The amount of cash to be received by Elton in addition to the machine before
any of the other partners received anything shall be equal to P15,000,
computed as follows:

Dalton Elton Falcon


Capital before liquidation P 40,000 P 90,000 P30,000
Divided by capital ratio 1/3 1/3 1/3_ Loss
absorption capacity P120,000 P270,000 P90,000

First priority payment to Elton:


1/3 x (P270,000 – P120,000) = P50,000

Less fair value of machine (35,000)


Amount of cash in first priority payment P15,000

74
. Letter “A” is the correct answer.
The amount of cash to be received by Elton in addition to the machine before
any of the other partners received anything shall be equal to P15,000,
computed as follows:

Dalton Elton Falcon


Capital before liquidation P 40,000 P 90,000 P30,000
Divided by capital ratio 1/3 1/3 1/3_ Loss
absorption capacity P120,000 P270,000 P90,000

First priority payment to Elton:


1/3 x (P270,000 – P120,000) = P50,000

Less fair value of machine (35,000)


Amount of cash in first priority payment P15,000

75
. Letter “A” is the correct answer.
The amount of cash to be received by Elton in addition to the machine before
any of the other partners received anything shall be equal to P15,000,
computed as follows:

Dalton Elton Falcon


Capital before liquidation P 40,000 P 90,000 P30,000
Divided by capital ratio 1/3 1/3 1/3_ Loss
absorption capacity P120,000 P270,000 P90,000

First priority payment to Elton:


1/3 x (P270,000 – P120,000) = P50,000

Less fair value of machine (35,000)


Amount of cash in first priority payment P15,000

76
. Letter “A” is the correct answer.
The amount of cash to be received by Elton in addition to the machine before
any of the other partners received anything shall be equal to P15,000,
computed as follows:

Dalton Elton Falcon


Capital before liquidation P 40,000 P 90,000 P30,000
Divided by capital ratio 1/3 1/3 1/3_ Loss
absorption capacity P120,000 P270,000 P90,000

First priority payment to Elton:


1/3 x (P270,000 – P120,000) = P50,000

Less fair value of machine (35,000)


Amount of cash in first priority payment P15,000

77
. Letter “A” is the correct answer.
The amount of cash to be received by Elton in addition to the machine before
any of the other partners received anything shall be equal to P15,000,
computed as follows:

Dalton Elton Falcon


Capital before liquidation P 40,000 P 90,000 P30,000
Divided by capital ratio 1/3 1/3 1/3_ Loss
absorption capacity P120,000 P270,000 P90,000

First priority payment to Elton:


1/3 x (P270,000 – P120,000) = P50,000

Less fair value of machine (35,000)


Amount of cash in first priority payment P15,000

78
. Letter “A” is the correct answer.
The amount of cash to be received by Elton in addition to the machine before
any of the other partners received anything shall be equal to P15,000,
computed as follows:

Dalton Elton Falcon


Capital before liquidation P 40,000 P 90,000 P30,000
Divided by capital ratio 1/3 1/3 1/3_ Loss
absorption capacity P120,000 P270,000 P90,000

First priority payment to Elton:


1/3 x (P270,000 – P120,000) = P50,000

Less fair value of machine (35,000)


Amount of cash in first priority payment P15,000

79
. Letter “A” is the correct answer.
The amount of cash to be received by Elton in addition to the machine before
any of the other partners received anything shall be equal to P15,000,
computed as follows:

Dalton Elton Falcon


Capital before liquidation P 40,000 P 90,000 P30,000
Divided by capital ratio 1/3 1/3 1/3_ Loss
absorption capacity P120,000 P270,000 P90,000

First priority payment to Elton:


1/3 x (P270,000 – P120,000) = P50,000

Less fair value of machine (35,000)


Amount of cash in first priority payment P15,000

80
. Letter “A” is the correct answer.
The amount of cash to be received by Elton in addition to the machine before
any of the other partners received anything shall be equal to P15,000,
computed as follows:

Dalton Elton Falcon


Capital before liquidation P 40,000 P 90,000 P30,000
Divided by capital ratio 1/3 1/3 1/3_ Loss
absorption capacity P120,000 P270,000 P90,000

First priority payment to Elton:


1/3 x (P270,000 – P120,000) = P50,000

Less fair value of machine (35,000)


Amount of cash in first priority payment P15,000

81
. Letter “A” is the correct answer.
The amount of cash to be received by Elton in addition to the machine before
any of the other partners received anything shall be equal to P15,000,
computed as follows:

Dalton Elton Falcon


Capital before liquidation P 40,000 P 90,000 P30,000
Divided by capital ratio 1/3 1/3 1/3_ Loss
absorption capacity P120,000 P270,000 P90,000

First priority payment to Elton:


1/3 x (P270,000 – P120,000) = P50,000

Less fair value of machine (35,000)


Amount of cash in first priority payment P15,000

82
. Letter “A” is the correct answer.
The amount of cash to be received by Elton in addition to the machine before
any of the other partners received anything shall be equal to P15,000,
computed as follows:

Dalton Elton Falcon


Capital before liquidation P 40,000 P 90,000 P30,000
Divided by capital ratio 1/3 1/3 1/3_ Loss
absorption capacity P120,000 P270,000 P90,000

First priority payment to Elton:


1/3 x (P270,000 – P120,000) = P50,000

Less fair value of machine (35,000)


Amount of cash in first priority payment P15,000

83
. Letter “A” is the correct answer.
The amount of cash to be received by Elton in addition to the machine before
any of the other partners received anything shall be equal to P15,000,
computed as follows:

Dalton Elton Falcon


Capital before liquidation P 40,000 P 90,000 P30,000
Divided by capital ratio 1/3 1/3 1/3_ Loss
absorption capacity P120,000 P270,000 P90,000

First priority payment to Elton:


1/3 x (P270,000 – P120,000) = P50,000

Less fair value of machine (35,000)


Amount of cash in first priority payment P15,000

84
. Letter “A” is the correct answer.
The amount of cash to be received by Elton in addition to the machine before
any of the other partners received anything shall be equal to P15,000,
computed as follows:

Dalton Elton Falcon


Capital before liquidation P 40,000 P 90,000 P30,000
Divided by capital ratio 1/3 1/3 1/3_ Loss
absorption capacity P120,000 P270,000 P90,000

First priority payment to Elton:


1/3 x (P270,000 – P120,000) = P50,000

Less fair value of machine (35,000)


Amount of cash in first priority payment P15,000

85
. Letter “A” is the correct answer.
The amount of cash to be received by Elton in addition to the machine before
any of the other partners received anything shall be equal to P15,000,
computed as follows:

Dalton Elton Falcon


Capital before liquidation P 40,000 P 90,000 P30,000
Divided by capital ratio 1/3 1/3 1/3_ Loss
absorption capacity P120,000 P270,000 P90,000

First priority payment to Elton:


1/3 x (P270,000 – P120,000) = P50,000

Less fair value of machine (35,000)


Amount of cash in first priority payment P15,000

86
. Letter “A” is the correct answer.
The amount of cash to be received by Elton in addition to the machine before
any of the other partners received anything shall be equal to P15,000,
computed as follows:

Dalton Elton Falcon


Capital before liquidation P 40,000 P 90,000 P30,000
Divided by capital ratio 1/3 1/3 1/3_ Loss
absorption capacity P120,000 P270,000 P90,000

First priority payment to Elton:


1/3 x (P270,000 – P120,000) = P50,000

Less fair value of machine (35,000)


Amount of cash in first priority payment P15,000

87
. Letter “A” is the correct answer.
The amount of cash to be received by Elton in addition to the machine before
any of the other partners received anything shall be equal to P15,000,
computed as follows:

Dalton Elton Falcon


Capital before liquidation P 40,000 P 90,000 P30,000
Divided by capital ratio 1/3 1/3 1/3_ Loss
absorption capacity P120,000 P270,000 P90,000

First priority payment to Elton:


1/3 x (P270,000 – P120,000) = P50,000

Less fair value of machine (35,000)


Amount of cash in first priority payment P15,000

88
. Letter “A” is the correct answer.
The amount of cash to be received by Elton in addition to the machine before
any of the other partners received anything shall be equal to P15,000,
computed as follows:

Dalton Elton Falcon


Capital before liquidation P 40,000 P 90,000 P30,000
Divided by capital ratio 1/3 1/3 1/3_ Loss
absorption capacity P120,000 P270,000 P90,000

First priority payment to Elton:


1/3 x (P270,000 – P120,000) = P50,000

Less fair value of machine (35,000)


Amount of cash in first priority payment P15,000

89
. Letter “A” is the correct answer.
The amount of cash to be received by Elton in addition to the machine before
any of the other partners received anything shall be equal to P15,000,
computed as follows:

Dalton Elton Falcon


Capital before liquidation P 40,000 P 90,000 P30,000
Divided by capital ratio 1/3 1/3 1/3_ Loss
absorption capacity P120,000 P270,000 P90,000

First priority payment to Elton:


1/3 x (P270,000 – P120,000) = P50,000

Less fair value of machine (35,000)


Amount of cash in first priority payment P15,000

90
. Letter “A” is the correct answer.
The amount of cash to be received by Elton in addition to the machine before
any of the other partners received anything shall be equal to P15,000,
computed as follows:

Dalton Elton Falcon


Capital before liquidation P 40,000 P 90,000 P30,000
Divided by capital ratio 1/3 1/3 1/3_ Loss
absorption capacity P120,000 P270,000 P90,000

First priority payment to Elton:


1/3 x (P270,000 – P120,000) = P50,000

Less fair value of machine (35,000)


Amount of cash in first priority payment P15,000

91
. Letter “A” is the correct answer.
The amount of cash to be received by Elton in addition to the machine before
any of the other partners received anything shall be equal to P15,000,
computed as follows:

Dalton Elton Falcon


Capital before liquidation P 40,000 P 90,000 P30,000
Divided by capital ratio 1/3 1/3 1/3_ Loss
absorption capacity P120,000 P270,000 P90,000

First priority payment to Elton:


1/3 x (P270,000 – P120,000) = P50,000

Less fair value of machine (35,000)


Amount of cash in first priority payment P15,000

92
. Letter “A” is the correct answer.
The amount of cash to be received by Elton in addition to the machine before
any of the other partners received anything shall be equal to P15,000,
computed as follows:

Dalton Elton Falcon


Capital before liquidation P 40,000 P 90,000 P30,000
Divided by capital ratio 1/3 1/3 1/3_ Loss
absorption capacity P120,000 P270,000 P90,000

First priority payment to Elton:


1/3 x (P270,000 – P120,000) = P50,000

Less fair value of machine (35,000)


Amount of cash in first priority payment P15,000

93
. Letter “A” is the correct answer.
The amount of cash to be received by Elton in addition to the machine before
any of the other partners received anything shall be equal to P15,000,
computed as follows:

Dalton Elton Falcon


Capital before liquidation P 40,000 P 90,000 P30,000
Divided by capital ratio 1/3 1/3 1/3_ Loss
absorption capacity P120,000 P270,000 P90,000

First priority payment to Elton:


1/3 x (P270,000 – P120,000) = P50,000

Less fair value of machine (35,000)


Amount of cash in first priority payment P15,000

94
. Letter “A” is the correct answer.
The amount of cash to be received by Elton in addition to the machine before
any of the other partners received anything shall be equal to P15,000,
computed as follows:

Dalton Elton Falcon


Capital before liquidation P 40,000 P 90,000 P30,000
Divided by capital ratio 1/3 1/3 1/3_ Loss
absorption capacity P120,000 P270,000 P90,000

First priority payment to Elton:


1/3 x (P270,000 – P120,000) = P50,000

Less fair value of machine (35,000)


Amount of cash in first priority payment P15,000

95
. Letter “A” is the correct answer.
The amount of cash to be received by Elton in addition to the machine before
any of the other partners received anything shall be equal to P15,000,
computed as follows:

Dalton Elton Falcon


Capital before liquidation P 40,000 P 90,000 P30,000
Divided by capital ratio 1/3 1/3 1/3_ Loss
absorption capacity P120,000 P270,000 P90,000

First priority payment to Elton:


1/3 x (P270,000 – P120,000) = P50,000

Less fair value of machine (35,000)


Amount of cash in first priority payment P15,000

96
. Letter “A” is the correct answer.
The amount of cash to be received by Elton in addition to the machine before
any of the other partners received anything shall be equal to P15,000,
computed as follows:

Dalton Elton Falcon


Capital before liquidation P 40,000 P 90,000 P30,000
Divided by capital ratio 1/3 1/3 1/3_ Loss
absorption capacity P120,000 P270,000 P90,000

First priority payment to Elton:


1/3 x (P270,000 – P120,000) = P50,000

Less fair value of machine (35,000)


Amount of cash in first priority payment P15,000

97
. Letter “A” is the correct answer.
The amount of cash to be received by Elton in addition to the machine before
any of the other partners received anything shall be equal to P15,000,
computed as follows:

Dalton Elton Falcon


Capital before liquidation P 40,000 P 90,000 P30,000
Divided by capital ratio 1/3 1/3 1/3_ Loss
absorption capacity P120,000 P270,000 P90,000

First priority payment to Elton:


1/3 x (P270,000 – P120,000) = P50,000

Less fair value of machine (35,000)


Amount of cash in first priority payment P15,000

98
. Letter “A” is the correct answer.
The amount of cash to be received by Elton in addition to the machine before
any of the other partners received anything shall be equal to P15,000,
computed as follows:

Dalton Elton Falcon


Capital before liquidation P 40,000 P 90,000 P30,000
Divided by capital ratio 1/3 1/3 1/3_ Loss
absorption capacity P120,000 P270,000 P90,000

First priority payment to Elton:


1/3 x (P270,000 – P120,000) = P50,000

Less fair value of machine (35,000)


Amount of cash in first priority payment P15,000

99
. Letter “A” is the correct answer.
The amount of cash to be received by Elton in addition to the machine before
any of the other partners received anything shall be equal to P15,000,
computed as follows:

Dalton Elton Falcon


Capital before liquidation P 40,000 P 90,000 P30,000
Divided by capital ratio 1/3 1/3 1/3_ Loss
absorption capacity P120,000 P270,000 P90,000

First priority payment to Elton:


1/3 x (P270,000 – P120,000) = P50,000

Less fair value of machine (35,000)


Amount of cash in first priority payment P15,000

100
. Letter “A” is the correct answer.
The amount of cash to be received by Elton in addition to the machine before
any of the other partners received anything shall be equal to P15,000,
computed as follows:

Dalton Elton Falcon


Capital before liquidation P 40,000 P 90,000 P30,000
Divided by capital ratio 1/3 1/3 1/3_ Loss
absorption capacity P120,000 P270,000 P90,000

First priority payment to Elton:


1/3 x (P270,000 – P120,000) = P50,000

Less fair value of machine (35,000)


Amount of cash in first priority payment P15,000

101
. Letter “A” is the correct answer.
The amount of cash to be received by Elton in addition to the machine before
any of the other partners received anything shall be equal to P15,000,
computed as follows:

Dalton Elton Falcon


Capital before liquidation P 40,000 P 90,000 P30,000
Divided by capital ratio 1/3 1/3 1/3_ Loss
absorption capacity P120,000 P270,000 P90,000

First priority payment to Elton:


1/3 x (P270,000 – P120,000) = P50,000

Less fair value of machine (35,000)


Amount of cash in first priority payment P15,000

102
. Letter “A” is the correct answer.
The amount of cash to be received by Elton in addition to the machine before
any of the other partners received anything shall be equal to P15,000,
computed as follows:

Dalton Elton Falcon


Capital before liquidation P 40,000 P 90,000 P30,000
Divided by capital ratio 1/3 1/3 1/3_ Loss
absorption capacity P120,000 P270,000 P90,000

First priority payment to Elton:


1/3 x (P270,000 – P120,000) = P50,000

Less fair value of machine (35,000)


Amount of cash in first priority payment P15,000

103
. Letter “A” is the correct answer.
The amount of cash to be received by Elton in addition to the machine before
any of the other partners received anything shall be equal to P15,000,
computed as follows:

Dalton Elton Falcon


Capital before liquidation P 40,000 P 90,000 P30,000
Divided by capital ratio 1/3 1/3 1/3_ Loss
absorption capacity P120,000 P270,000 P90,000

First priority payment to Elton:


1/3 x (P270,000 – P120,000) = P50,000

Less fair value of machine (35,000)


Amount of cash in first priority payment P15,000

104
. Letter “A” is the correct answer.
The amount of cash to be received by Elton in addition to the machine before
any of the other partners received anything shall be equal to P15,000,
computed as follows:

Dalton Elton Falcon


Capital before liquidation P 40,000 P 90,000 P30,000
Divided by capital ratio 1/3 1/3 1/3_ Loss
absorption capacity P120,000 P270,000 P90,000

First priority payment to Elton:


1/3 x (P270,000 – P120,000) = P50,000

Less fair value of machine (35,000)


Amount of cash in first priority payment P15,000

105
. Letter “A” is the correct answer.
The amount of cash to be received by Elton in addition to the machine before
any of the other partners received anything shall be equal to P15,000,
computed as follows:

Dalton Elton Falcon


Capital before liquidation P 40,000 P 90,000 P30,000
Divided by capital ratio 1/3 1/3 1/3_ Loss
absorption capacity P120,000 P270,000 P90,000

First priority payment to Elton:


1/3 x (P270,000 – P120,000) = P50,000

Less fair value of machine (35,000)


Amount of cash in first priority payment P15,000

106
. Letter “A” is the correct answer.
The amount of cash to be received by Elton in addition to the machine before
any of the other partners received anything shall be equal to P15,000,
computed as follows:

Dalton Elton Falcon


Capital before liquidation P 40,000 P 90,000 P30,000
Divided by capital ratio 1/3 1/3 1/3_ Loss
absorption capacity P120,000 P270,000 P90,000

First priority payment to Elton:


1/3 x (P270,000 – P120,000) = P50,000

Less fair value of machine (35,000)


Amount of cash in first priority payment P15,000

107
. Letter “A” is the correct answer.
The amount of cash to be received by Elton in addition to the machine before
any of the other partners received anything shall be equal to P15,000,
computed as follows:

Dalton Elton Falcon


Capital before liquidation P 40,000 P 90,000 P30,000
Divided by capital ratio 1/3 1/3 1/3_ Loss
absorption capacity P120,000 P270,000 P90,000

First priority payment to Elton:


1/3 x (P270,000 – P120,000) = P50,000

Less fair value of machine (35,000)


Amount of cash in first priority payment P15,000

108
. Letter “A” is the correct answer.
The amount of cash to be received by Elton in addition to the machine before
any of the other partners received anything shall be equal to P15,000,
computed as follows:

Dalton Elton Falcon


Capital before liquidation P 40,000 P 90,000 P30,000
Divided by capital ratio 1/3 1/3 1/3_ Loss
absorption capacity P120,000 P270,000 P90,000

First priority payment to Elton:


1/3 x (P270,000 – P120,000) = P50,000

Less fair value of machine (35,000)


Amount of cash in first priority payment P15,000

109
. Letter “A” is the correct answer.
The amount of cash to be received by Elton in addition to the machine before
any of the other partners received anything shall be equal to P15,000,
computed as follows:

Dalton Elton Falcon


Capital before liquidation P 40,000 P 90,000 P30,000
Divided by capital ratio 1/3 1/3 1/3_ Loss
absorption capacity P120,000 P270,000 P90,000

First priority payment to Elton:


1/3 x (P270,000 – P120,000) = P50,000

Less fair value of machine (35,000)


Amount of cash in first priority payment P15,000

110
. Letter “A” is the correct answer.
The amount of cash to be received by Elton in addition to the machine before
any of the other partners received anything shall be equal to P15,000,
computed as follows:

Dalton Elton Falcon


Capital before liquidation P 40,000 P 90,000 P30,000
Divided by capital ratio 1/3 1/3 1/3_ Loss
absorption capacity P120,000 P270,000 P90,000

First priority payment to Elton:


1/3 x (P270,000 – P120,000) = P50,000

Less fair value of machine (35,000)


Amount of cash in first priority payment P15,000

111
. Letter “A” is the correct answer.
The amount of cash to be received by Elton in addition to the machine before
any of the other partners received anything shall be equal to P15,000,
computed as follows:

Dalton Elton Falcon


Capital before liquidation P 40,000 P 90,000 P30,000
Divided by capital ratio 1/3 1/3 1/3_ Loss
absorption capacity P120,000 P270,000 P90,000

First priority payment to Elton:


1/3 x (P270,000 – P120,000) = P50,000

Less fair value of machine (35,000)


Amount of cash in first priority payment P15,000

112
. Letter “A” is the correct answer.
The amount of cash to be received by Elton in addition to the machine before
any of the other partners received anything shall be equal to P15,000,
computed as follows:

Dalton Elton Falcon


Capital before liquidation P 40,000 P 90,000 P30,000
Divided by capital ratio 1/3 1/3 1/3_ Loss
absorption capacity P120,000 P270,000 P90,000

First priority payment to Elton:


1/3 x (P270,000 – P120,000) = P50,000

Less fair value of machine (35,000)


Amount of cash in first priority payment P15,000

113
. Letter “A” is the correct answer.
The amount of cash to be received by Elton in addition to the machine before
any of the other partners received anything shall be equal to P15,000,
computed as follows:

Dalton Elton Falcon


Capital before liquidation P 40,000 P 90,000 P30,000
Divided by capital ratio 1/3 1/3 1/3_ Loss
absorption capacity P120,000 P270,000 P90,000

First priority payment to Elton:


1/3 x (P270,000 – P120,000) = P50,000

Less fair value of machine (35,000)


Amount of cash in first priority payment P15,000

114
. Letter “A” is the correct answer.
The amount of cash to be received by Elton in addition to the machine before
any of the other partners received anything shall be equal to P15,000,
computed as follows:

Dalton Elton Falcon


Capital before liquidation P 40,000 P 90,000 P30,000
Divided by capital ratio 1/3 1/3 1/3_ Loss
absorption capacity P120,000 P270,000 P90,000

First priority payment to Elton:


1/3 x (P270,000 – P120,000) = P50,000

Less fair value of machine (35,000)


Amount of cash in first priority payment P15,000

115
. Letter “A” is the correct answer.
The amount of cash to be received by Elton in addition to the machine before
any of the other partners received anything shall be equal to P15,000,
computed as follows:

Dalton Elton Falcon


Capital before liquidation P 40,000 P 90,000 P30,000
Divided by capital ratio 1/3 1/3 1/3_ Loss
absorption capacity P120,000 P270,000 P90,000

First priority payment to Elton:


1/3 x (P270,000 – P120,000) = P50,000

Less fair value of machine (35,000)


Amount of cash in first priority payment P15,000

116
. Letter “A” is the correct answer.
The amount of cash to be received by Elton in addition to the machine before
any of the other partners received anything shall be equal to P15,000,
computed as follows:

Dalton Elton Falcon


Capital before liquidation P 40,000 P 90,000 P30,000
Divided by capital ratio 1/3 1/3 1/3_ Loss
absorption capacity P120,000 P270,000 P90,000

First priority payment to Elton:


1/3 x (P270,000 – P120,000) = P50,000

Less fair value of machine (35,000)


Amount of cash in first priority payment P15,000

117
. Letter “A” is the correct answer.
The amount of cash to be received by Elton in addition to the machine before
any of the other partners received anything shall be equal to P15,000,
computed as follows:

Dalton Elton Falcon


Capital before liquidation P 40,000 P 90,000 P30,000
Divided by capital ratio 1/3 1/3 1/3_ Loss
absorption capacity P120,000 P270,000 P90,000

First priority payment to Elton:


1/3 x (P270,000 – P120,000) = P50,000

Less fair value of machine (35,000)


Amount of cash in first priority payment P15,000

118
. Letter “A” is the correct answer.
The amount of cash to be received by Elton in addition to the machine before
any of the other partners received anything shall be equal to P15,000,
computed as follows:

Dalton Elton Falcon


Capital before liquidation P 40,000 P 90,000 P30,000
Divided by capital ratio 1/3 1/3 1/3_ Loss
absorption capacity P120,000 P270,000 P90,000

First priority payment to Elton:


1/3 x (P270,000 – P120,000) = P50,000

Less fair value of machine (35,000)


Amount of cash in first priority payment P15,000

119
. Letter “A” is the correct answer.
The amount of cash to be received by Elton in addition to the machine before
any of the other partners received anything shall be equal to P15,000,
computed as follows:

Dalton Elton Falcon


Capital before liquidation P 40,000 P 90,000 P30,000
Divided by capital ratio 1/3 1/3 1/3_ Loss
absorption capacity P120,000 P270,000 P90,000

First priority payment to Elton:


1/3 x (P270,000 – P120,000) = P50,000

Less fair value of machine (35,000)


Amount of cash in first priority payment P15,000

120
. Letter “A” is the correct answer.
The amount of cash to be received by Elton in addition to the machine before
any of the other partners received anything shall be equal to P15,000,
computed as follows:

Dalton Elton Falcon


Capital before liquidation P 40,000 P 90,000 P30,000
Divided by capital ratio 1/3 1/3 1/3_ Loss
absorption capacity P120,000 P270,000 P90,000

First priority payment to Elton:


1/3 x (P270,000 – P120,000) = P50,000

Less fair value of machine (35,000)


Amount of cash in first priority payment P15,000

121
. Letter “A” is the correct answer.
The amount of cash to be received by Elton in addition to the machine before
any of the other partners received anything shall be equal to P15,000,
computed as follows:

Dalton Elton Falcon


Capital before liquidation P 40,000 P 90,000 P30,000
Divided by capital ratio 1/3 1/3 1/3_ Loss
absorption capacity P120,000 P270,000 P90,000

First priority payment to Elton:


1/3 x (P270,000 – P120,000) = P50,000

Less fair value of machine (35,000)


Amount of cash in first priority payment P15,000

122
. Letter “A” is the correct answer.
The amount of cash to be received by Elton in addition to the machine before
any of the other partners received anything shall be equal to P15,000,
computed as follows:

Dalton Elton Falcon


Capital before liquidation P 40,000 P 90,000 P30,000
Divided by capital ratio 1/3 1/3 1/3_ Loss
absorption capacity P120,000 P270,000 P90,000

First priority payment to Elton:


1/3 x (P270,000 – P120,000) = P50,000

Less fair value of machine (35,000)


Amount of cash in first priority payment P15,000

123
. Letter “A” is the correct answer.
The amount of cash to be received by Elton in addition to the machine before
any of the other partners received anything shall be equal to P15,000,
computed as follows:

Dalton Elton Falcon


Capital before liquidation P 40,000 P 90,000 P30,000
Divided by capital ratio 1/3 1/3 1/3_ Loss
absorption capacity P120,000 P270,000 P90,000

First priority payment to Elton:


1/3 x (P270,000 – P120,000) = P50,000

Less fair value of machine (35,000)


Amount of cash in first priority payment P15,000

124
. Letter “A” is the correct answer.
The amount of cash to be received by Elton in addition to the machine before
any of the other partners received anything shall be equal to P15,000,
computed as follows:

Dalton Elton Falcon


Capital before liquidation P 40,000 P 90,000 P30,000
Divided by capital ratio 1/3 1/3 1/3_ Loss
absorption capacity P120,000 P270,000 P90,000

First priority payment to Elton:


1/3 x (P270,000 – P120,000) = P50,000

Less fair value of machine (35,000)


Amount of cash in first priority payment P15,000

125
. Letter “A” is the correct answer.
The amount of cash to be received by Elton in addition to the machine before
any of the other partners received anything shall be equal to P15,000,
computed as follows:

Dalton Elton Falcon


Capital before liquidation P 40,000 P 90,000 P30,000
Divided by capital ratio 1/3 1/3 1/3_ Loss
absorption capacity P120,000 P270,000 P90,000

First priority payment to Elton:


1/3 x (P270,000 – P120,000) = P50,000

Less fair value of machine (35,000)


Amount of cash in first priority payment P15,000

126
. Letter “A” is the correct answer.
The amount of cash to be received by Elton in addition to the machine before
any of the other partners received anything shall be equal to P15,000,
computed as follows:

Dalton Elton Falcon


Capital before liquidation P 40,000 P 90,000 P30,000
Divided by capital ratio 1/3 1/3 1/3_ Loss
absorption capacity P120,000 P270,000 P90,000

First priority payment to Elton:


1/3 x (P270,000 – P120,000) = P50,000

Less fair value of machine (35,000)


Amount of cash in first priority payment P15,000

127
. Letter “A” is the correct answer.
The amount of cash to be received by Elton in addition to the machine before
any of the other partners received anything shall be equal to P15,000,
computed as follows:

Dalton Elton Falcon


Capital before liquidation P 40,000 P 90,000 P30,000
Divided by capital ratio 1/3 1/3 1/3_ Loss
absorption capacity P120,000 P270,000 P90,000

First priority payment to Elton:


1/3 x (P270,000 – P120,000) = P50,000

Less fair value of machine (35,000)


Amount of cash in first priority payment P15,000

128
. Letter “A” is the correct answer.
The amount of cash to be received by Elton in addition to the machine before
any of the other partners received anything shall be equal to P15,000,
computed as follows:

Dalton Elton Falcon


Capital before liquidation P 40,000 P 90,000 P30,000
Divided by capital ratio 1/3 1/3 1/3_ Loss
absorption capacity P120,000 P270,000 P90,000

First priority payment to Elton:


1/3 x (P270,000 – P120,000) = P50,000

Less fair value of machine (35,000)


Amount of cash in first priority payment P15,000

129
. Letter “A” is the correct answer.
The amount of cash to be received by Elton in addition to the machine before
any of the other partners received anything shall be equal to P15,000,
computed as follows:

Dalton Elton Falcon


Capital before liquidation P 40,000 P 90,000 P30,000
Divided by capital ratio 1/3 1/3 1/3_ Loss
absorption capacity P120,000 P270,000 P90,000

First priority payment to Elton:


1/3 x (P270,000 – P120,000) = P50,000

Less fair value of machine (35,000)


Amount of cash in first priority payment P15,000

130
. Letter “A” is the correct answer.
The amount of cash to be received by Elton in addition to the machine before
any of the other partners received anything shall be equal to P15,000,
computed as follows:

Dalton Elton Falcon


Capital before liquidation P 40,000 P 90,000 P30,000
Divided by capital ratio 1/3 1/3 1/3_ Loss
absorption capacity P120,000 P270,000 P90,000

First priority payment to Elton:


1/3 x (P270,000 – P120,000) = P50,000

Less fair value of machine (35,000)


Amount of cash in first priority payment P15,000

131
. Letter “A” is the correct answer.
The amount of cash to be received by Elton in addition to the machine before
any of the other partners received anything shall be equal to P15,000,
computed as follows:

Dalton Elton Falcon


Capital before liquidation P 40,000 P 90,000 P30,000
Divided by capital ratio 1/3 1/3 1/3_ Loss
absorption capacity P120,000 P270,000 P90,000

First priority payment to Elton:


1/3 x (P270,000 – P120,000) = P50,000

Less fair value of machine (35,000)


Amount of cash in first priority payment P15,000

132
. Letter “A” is the correct answer.
The amount of cash to be received by Elton in addition to the machine before
any of the other partners received anything shall be equal to P15,000,
computed as follows:

Dalton Elton Falcon


Capital before liquidation P 40,000 P 90,000 P30,000
Divided by capital ratio 1/3 1/3 1/3_ Loss
absorption capacity P120,000 P270,000 P90,000

First priority payment to Elton:


1/3 x (P270,000 – P120,000) = P50,000

Less fair value of machine (35,000)


Amount of cash in first priority payment P15,000

133
. Letter “A” is the correct answer.
The amount of cash to be received by Elton in addition to the machine before
any of the other partners received anything shall be equal to P15,000,
computed as follows:

Dalton Elton Falcon


Capital before liquidation P 40,000 P 90,000 P30,000
Divided by capital ratio 1/3 1/3 1/3_ Loss
absorption capacity P120,000 P270,000 P90,000

First priority payment to Elton:


1/3 x (P270,000 – P120,000) = P50,000

Less fair value of machine (35,000)


Amount of cash in first priority payment P15,000

134
. Letter “A” is the correct answer.
The amount of cash to be received by Elton in addition to the machine before
any of the other partners received anything shall be equal to P15,000,
computed as follows:

Dalton Elton Falcon


Capital before liquidation P 40,000 P 90,000 P30,000
Divided by capital ratio 1/3 1/3 1/3_ Loss
absorption capacity P120,000 P270,000 P90,000

First priority payment to Elton:


1/3 x (P270,000 – P120,000) = P50,000

Less fair value of machine (35,000)


Amount of cash in first priority payment P15,000

135
. Letter “A” is the correct answer.
The amount of cash to be received by Elton in addition to the machine before
any of the other partners received anything shall be equal to P15,000,
computed as follows:

Dalton Elton Falcon


Capital before liquidation P 40,000 P 90,000 P30,000
Divided by capital ratio 1/3 1/3 1/3_ Loss
absorption capacity P120,000 P270,000 P90,000

First priority payment to Elton:


1/3 x (P270,000 – P120,000) = P50,000

Less fair value of machine (35,000)


Amount of cash in first priority payment P15,000

136
. Letter “A” is the correct answer.
The amount of cash to be received by Elton in addition to the machine before
any of the other partners received anything shall be equal to P15,000,
computed as follows:

Dalton Elton Falcon


Capital before liquidation P 40,000 P 90,000 P30,000
Divided by capital ratio 1/3 1/3 1/3_ Loss
absorption capacity P120,000 P270,000 P90,000

First priority payment to Elton:


1/3 x (P270,000 – P120,000) = P50,000

Less fair value of machine (35,000)


Amount of cash in first priority payment P15,000

137
. Letter “A” is the correct answer.
The amount of cash to be received by Elton in addition to the machine before
any of the other partners received anything shall be equal to P15,000,
computed as follows:

Dalton Elton Falcon


Capital before liquidation P 40,000 P 90,000 P30,000
Divided by capital ratio 1/3 1/3 1/3_ Loss
absorption capacity P120,000 P270,000 P90,000

First priority payment to Elton:


1/3 x (P270,000 – P120,000) = P50,000

Less fair value of machine (35,000)


Amount of cash in first priority payment P15,000

138
. Letter “A” is the correct answer.
The amount of cash to be received by Elton in addition to the machine before
any of the other partners received anything shall be equal to P15,000,
computed as follows:

Dalton Elton Falcon


Capital before liquidation P 40,000 P 90,000 P30,000
Divided by capital ratio 1/3 1/3 1/3_ Loss
absorption capacity P120,000 P270,000 P90,000

First priority payment to Elton:


1/3 x (P270,000 – P120,000) = P50,000

Less fair value of machine (35,000)


Amount of cash in first priority payment P15,000

139
. Letter “A” is the correct answer.
The amount of cash to be received by Elton in addition to the machine before
any of the other partners received anything shall be equal to P15,000,
computed as follows:

Dalton Elton Falcon


Capital before liquidation P 40,000 P 90,000 P30,000
Divided by capital ratio 1/3 1/3 1/3_ Loss
absorption capacity P120,000 P270,000 P90,000

First priority payment to Elton:


1/3 x (P270,000 – P120,000) = P50,000

Less fair value of machine (35,000)


Amount of cash in first priority payment P15,000

140
. Letter “A” is the correct answer.
The amount of cash to be received by Elton in addition to the machine before
any of the other partners received anything shall be equal to P15,000,
computed as follows:

Dalton Elton Falcon


Capital before liquidation P 40,000 P 90,000 P30,000
Divided by capital ratio 1/3 1/3 1/3_ Loss
absorption capacity P120,000 P270,000 P90,000

First priority payment to Elton:


1/3 x (P270,000 – P120,000) = P50,000

Less fair value of machine (35,000)


Amount of cash in first priority payment P15,000

141
. Letter “A” is the correct answer.
The amount of cash to be received by Elton in addition to the machine before
any of the other partners received anything shall be equal to P15,000,
computed as follows:

Dalton Elton Falcon


Capital before liquidation P 40,000 P 90,000 P30,000
Divided by capital ratio 1/3 1/3 1/3_ Loss
absorption capacity P120,000 P270,000 P90,000

First priority payment to Elton:


1/3 x (P270,000 – P120,000) = P50,000

Less fair value of machine (35,000)


Amount of cash in first priority payment P15,000

142
. Letter “A” is the correct answer.
The amount of cash to be received by Elton in addition to the machine before
any of the other partners received anything shall be equal to P15,000,
computed as follows:

Dalton Elton Falcon


Capital before liquidation P 40,000 P 90,000 P30,000
Divided by capital ratio 1/3 1/3 1/3_ Loss
absorption capacity P120,000 P270,000 P90,000

First priority payment to Elton:


1/3 x (P270,000 – P120,000) = P50,000

Less fair value of machine (35,000)


Amount of cash in first priority payment P15,000

143
. Letter “A” is the correct answer.
The amount of cash to be received by Elton in addition to the machine before
any of the other partners received anything shall be equal to P15,000,
computed as follows:

Dalton Elton Falcon


Capital before liquidation P 40,000 P 90,000 P30,000
Divided by capital ratio 1/3 1/3 1/3_ Loss
absorption capacity P120,000 P270,000 P90,000

First priority payment to Elton:


1/3 x (P270,000 – P120,000) = P50,000

Less fair value of machine (35,000)


Amount of cash in first priority payment P15,000

144
. Letter “A” is the correct answer.
The amount of cash to be received by Elton in addition to the machine before
any of the other partners received anything shall be equal to P15,000,
computed as follows:

Dalton Elton Falcon


Capital before liquidation P 40,000 P 90,000 P30,000
Divided by capital ratio 1/3 1/3 1/3_ Loss
absorption capacity P120,000 P270,000 P90,000

First priority payment to Elton:


1/3 x (P270,000 – P120,000) = P50,000

Less fair value of machine (35,000)


Amount of cash in first priority payment P15,000

145
. Letter “A” is the correct answer.
The amount of cash to be received by Elton in addition to the machine before
any of the other partners received anything shall be equal to P15,000,
computed as follows:

Dalton Elton Falcon


Capital before liquidation P 40,000 P 90,000 P30,000
Divided by capital ratio 1/3 1/3 1/3_ Loss
absorption capacity P120,000 P270,000 P90,000

First priority payment to Elton:


1/3 x (P270,000 – P120,000) = P50,000

Less fair value of machine (35,000)


Amount of cash in first priority payment P15,000

146
. Letter “A” is the correct answer.
The amount of cash to be received by Elton in addition to the machine before
any of the other partners received anything shall be equal to P15,000,
computed as follows:

Dalton Elton Falcon


Capital before liquidation P 40,000 P 90,000 P30,000
Divided by capital ratio 1/3 1/3 1/3_ Loss
absorption capacity P120,000 P270,000 P90,000

First priority payment to Elton:


1/3 x (P270,000 – P120,000) = P50,000

Less fair value of machine (35,000)


Amount of cash in first priority payment P15,000

147
. Letter “A” is the correct answer.
The amount of cash to be received by Elton in addition to the machine before
any of the other partners received anything shall be equal to P15,000,
computed as follows:

Dalton Elton Falcon


Capital before liquidation P 40,000 P 90,000 P30,000
Divided by capital ratio 1/3 1/3 1/3_ Loss
absorption capacity P120,000 P270,000 P90,000

First priority payment to Elton:


1/3 x (P270,000 – P120,000) = P50,000

Less fair value of machine (35,000)


Amount of cash in first priority payment P15,000

148
. Letter “A” is the correct answer.
The amount of cash to be received by Elton in addition to the machine before
any of the other partners received anything shall be equal to P15,000,
computed as follows:

Dalton Elton Falcon


Capital before liquidation P 40,000 P 90,000 P30,000
Divided by capital ratio 1/3 1/3 1/3_ Loss
absorption capacity P120,000 P270,000 P90,000

First priority payment to Elton:


1/3 x (P270,000 – P120,000) = P50,000

Less fair value of machine (35,000)


Amount of cash in first priority payment P15,000

149
. Letter “A” is the correct answer.
The amount of cash to be received by Elton in addition to the machine before
any of the other partners received anything shall be equal to P15,000,
computed as follows:

Dalton Elton Falcon


Capital before liquidation P 40,000 P 90,000 P30,000
Divided by capital ratio 1/3 1/3 1/3_ Loss
absorption capacity P120,000 P270,000 P90,000

First priority payment to Elton:


1/3 x (P270,000 – P120,000) = P50,000

Less fair value of machine (35,000)


Amount of cash in first priority payment P15,000

150
. Letter “A” is the correct answer.
The amount of cash to be received by Elton in addition to the machine before
any of the other partners received anything shall be equal to P15,000,
computed as follows:

Dalton Elton Falcon


Capital before liquidation P 40,000 P 90,000 P30,000
Divided by capital ratio 1/3 1/3 1/3_ Loss
absorption capacity P120,000 P270,000 P90,000

First priority payment to Elton:


1/3 x (P270,000 – P120,000) = P50,000

Less fair value of machine (35,000)


Amount of cash in first priority payment P15,000

151
. Letter “A” is the correct answer.
The amount of cash to be received by Elton in addition to the machine before
any of the other partners received anything shall be equal to P15,000,
computed as follows:

Dalton Elton Falcon


Capital before liquidation P 40,000 P 90,000 P30,000
Divided by capital ratio 1/3 1/3 1/3_ Loss
absorption capacity P120,000 P270,000 P90,000

First priority payment to Elton:


1/3 x (P270,000 – P120,000) = P50,000

Less fair value of machine (35,000)


Amount of cash in first priority payment P15,000

You might also like

pFad - Phonifier reborn

Pfad - The Proxy pFad of © 2024 Garber Painting. All rights reserved.

Note: This service is not intended for secure transactions such as banking, social media, email, or purchasing. Use at your own risk. We assume no liability whatsoever for broken pages.


Alternative Proxies:

Alternative Proxy

pFad Proxy

pFad v3 Proxy

pFad v4 Proxy